Re: Ajuda para achar um livro

2001-09-07 Por tôpico Augusto Morgado

Ha uma ediçao recente da Dover que pode ser comprada sem grande
dificuldade.Sugiro a Internet (Amazon e Livraria Cultura) ou a Livraria
Kosmos, onde vi essa edição da Dover e onde se encontram muitos livros
da Dover. (www.kosmos.com.br)(0xx21 22248616)

Luis Lopes wrote:
> 
> Sauda,c~oes,
> 
> Será que alguém teria alguma coisa
> a dizer?
> 
> Um abraço,
> Luís
> 
> > -- Forwarded Message
> > From: "rodferro"<[EMAIL PROTECTED]>
> > Date: Wed,  5 Sep 2001 00:17:24 -0300
> > To: [EMAIL PROTECTED]
> > Subject: duvida-
> >
> > Caro professor,
> > estava lendo o livro Manual de progressoes,(que eh muito
> > bom ,parabens),quando eu deparei na bibliografia o
> > seguinte livro:
> > Shlarsky,D.O.,Chentzov,N.N,and Yaglom,I.M,The USSR
> > Olympiad Problem Book,W.H Freeman,1962.
> > Estou procurando por esse livro faz tempo,ja procurei em
> > sebo ,em varias bibliotecas,enfim muitos lugares.O
> > senhor saberia me informar onde posso compra-lo,ou
> > consulta-lo.(Pode ser tanto no rio ou em sp).
> > Muito Obrigado.
> > Rodrigo.
> > (Parabens mais uma vez pelo livro).
> > -- End of Forwarded Message
> >



Re: =?x-user-defined?q?Vari=E1veis?= discretas

2001-09-03 Por tôpico Augusto Morgado

Como o desvio-padrao nao se altera por translaçoes, o desvio eh o mesmo
se os valores fossem 0, 1,..., b-a. Chame b-a de p e os valores sao 0,
1,..., p. A variancia eh a media dos quadrados menos o quadrado da
media. A soma dos quadrados eh 1+4+...+p^2= p(p+1)(2p+1)/6. A media dos
quadrados eh p(2p+1)/6. A media eh p/2. A variancia eh p(2p+1)/6- pp/4 =
p(p+2)/12= ((p+1)^2-1)/12. O dp eh a raiz quadrada da variancia.

> Vinícius Damaso wrote:
> 
> Gostaria de uma ajuda:
>  
> Considere uma variável aleatória uniforme discreta que consiste de uma
> seqüência de inteiros consecutivos a, a+1, a+2,...,b com a<=b. A média
> é (b+a)/2. Mostre que o desvio padrão é sqrt(((b-a+1)^2-1)/12).
>  
> Desde já agradeço,
>  
> Vinícius Damaso.



Re: =?x-user-defined?q?Solu=E7=E3o?= dos problemas

2001-08-31 Por tôpico Augusto Morgado

Quem tem livros de enigmas publicados em portugues eh Raymond Smullian
(ou Smullyan, sei la). O mais famoso eh um que tem Alice no titulo. Ha
um outro, nao so de enigmas, que se chama a 1002-esima noite de
xerazade.

[EMAIL PROTECTED] wrote:
> 
>  Saudações a todos,
> 
>   Gostaria da ajuda de vcs para ter não só as respostas mas as
> soluções
> de alguns problemas que o prof. Paulo Santa Rita enviou para lista no
> dia
> 18/07/01 com o título " Problemas de Lógica ". Ele dizia que os
> problemas já
> haviam sido apresentados a lista por :
> 
> A) Bruno Leite
> problema das 5 casas de diferentes cores ... Quem tem peixe ?
> 
> B) Elon Santos Correa
> 1. problema do trem de 3 passageiros com os mesmos nome que os
> funcionários
> ... Quem é o foguista ?
> 2. problema das senhoras que compraram ... Que foi que cada uma
> comprou ?
> 
> C) Andre Luiz Arruda Marques
> problema da ilhas onde as pessoas mentem ou falam a verdade o dia
> inteiro ...
>  
>   Gostaria de pedir desculpas por retomar assuntos antigos mas eu
> não
> estava na lista quando essas soluções foram apresentadas. Gostaria
> também de
> indicações de bons livros ou páginas da internet sobre problemas de
> lógica.
>   Muito obrigado pela atenção,
> Raul



Re: Construção axiomática dos números

2001-08-12 Por tôpico Augusto Morgado

Um clássico é o Foundations of Analysis, de E. Landau (ou Landao?).
Morgado

David Daniel Turchick wrote:

> Será que alguém da lista poderia me sugerir um livro em que eu encontre uma
> construção axiomática dos números (em especial, dos conjuntos IN e IR)?
> 
> Obrigado,
> David
> 
> 
> 
> 
> 
> 
> 





Re: Berlin?

2001-08-07 Por tôpico Augusto Morgado

O Nicolau deve estar se referindo a ultima questao de multipla escolha.
Podia ser derivando, mas ha solucao mais simples transladando um dos
pontos rumo ao rio de uma distancia igual a largura do rio e tracando um
segmento de reta ate o outro ponto.
Morgado

Vinícius Damaso wrote:
> 
> Bem, sendo "d" a componente da distância XY perpendicular à margem do rio,
> "C" a projeção de X na sua margem mais próxima e "D" a projeção de Y na sua
> margem mais próxima temos que minimizar a função
> 
> F(CA)=(XC^2+CA^2)^.5 +AB+[(d-CA)^2+DY^2]^.5.
> 
> Aí deriva, iguala a zero e após alguma álgebra chega-se a
> 
> CA=d*XC/(XC+DY)
> 
> Era isso mesmo ou não podia ser derivando?
> 
> Um abraço,
> Damaso
> 
> - Original Message -
> From: Nicolau C. Saldanha <[EMAIL PROTECTED]>
> To: <[EMAIL PROTECTED]>
> Sent: Tuesday, August 07, 2001 4:45 PM
> Subject: Berlin?
> 
> > Vocês viram a última questão do último provão?
> > Não lembra nada a vocês? []s, N.
> >
> > PS: O dito provão está em
> > http://provao3.inep.gov.br/AvaliacaoIESPub/index.htm
> > http://www.mat.puc-rio.br/~nicolau/bobo/Provao.pdf
> >



Re: Bibliografia de Probabilidade =?x-user-defined?q?Geom=E9trica?=

2001-07-10 Por tôpico Augusto Morgado

Alguma coisa você encontra no volume 2 da Matemática do Ensino Médio.

> Edmilson wrote:
> 
> Caros amigos,
>  
> Eu estou precisando de material para um trabalho sobre probabilidade
> geométrica básica. Referências sobre os problemas : do Encontro, da
> agulha de Buffon, de áreas, e outros problemas.Pode ser  livros,
> periódicos ou páginas na Web.Um pouco de história também ajudaria.
>  
> Obrigado a todos.
>  
> Atenciosamente,
> Edmilson
> http://www.edmilsonaleixo.cjb.net



Re: =?x-user-defined?q?Quest=F5es?= de =?x-user-defined?q?combinat=F3ria=2Fjogos?=

2001-06-25 Por tôpico Augusto Morgado

É, mas o idiota aqui teria poupado muito esforço e teria sido muito mais
claro se tivesse começado escrevendo a+b=c como a=c-b.
Morgado

Paulo Santa Rita wrote:
> 
> Ola Pessoal,
> 
> A solucao abaixo - do Prof Morgado - e muito bonita ! A linha de raciocinio
> e muito semelhante a que leva a solucao de um outro problema olimpico, cujo
> enunciado segue abaixo :
> 
> Num poligono convexo de N lados,
> 
> 1)Dois lados quaisquer nao sao paralelos
> 2)Duas diagonais quaisquer nao sao paralelas
> 
> Quantos pontos no exterior do poligono sao pontos de inteseccao de diagonais
> ?
> 
> OBS : E dado que tres ou mais diagonais nunca se cruzam em um mesmo ponto.
> 
> >Leia a1 como a indice 1.
> >Observe inicialmente que a diferença entre dois elementos
> >distintos(maior-menor) do conjunto é ainda um relemento do conjunto.
> >Fixe a1=1. Considere as 48 diferenças a2-a1,..., a48-a1.Algum dos três
> >conjuntos conterá pelo menos dezesseis dessas 48 diferenças. Sejam b1,
> >b2,...,b16 essas diferenças e seja X o conjunto ao qual elas pertencem.
> >Considere as 15 diferenças b2-b1=c1,...,b16-b1=c15.
> >Se alguma dessa diferenças pertencer a X, X conterá b1, bk-b1 e bk, isto
> >é, as-a1, aj-a1-(as-a1)=aj-as e aj-a1; fim, pois o terceiro é a soma dos
> >dois primeiros.
> >
> >Caso contrário as 15 diferenças pertencerao aos outros dois conjuntos Y
> >e Z, havendo em um dos conjuntos, digamos Y, pelo menos 8 dessas
> >diferenças.Chamemos essas diferenças de d1,...,d8.Considere as 7
> >diferenças d2-d1,...,d8-d1.Note que essas diferenças sao diferenças
> >entre bês e portanto diferenças entre elementos da sequencia dos a,
> >estando ja excluida a possibilidade de alguma delas pertencer a X.
> >Se alguma dessa diferenças pertencer a Y, Y conterá d1, dp-d1 e dp, isto
> >é, bm-b1=ar-a1-(as-a1)=ar-as, bn-a1-(bm-b1)=bn-bm=(au-a1)-(ar-a1)=au-ar
> >e bn-b1=(au-a1)-(as-a1)=au-as; fim, pois o terceiro é a soma dos dois
> >primeiros.
> >Caso contrário, as 7 diferenças d2-d1=e1,...,d8-d1=e7 pertencerao a Z.
> >As seis diferenças e2-e1,...,e7-e1 pertencerao a Z pois sao diferenças
> >entre termos da sequencia dos d, estando ja excluida a possibilidade de
> >pertencerem a Y. Entao Z contera e1, ef-e1, ef ...fim, pois o terceiro é
> >a soma dos dois primeiros.
> >
> >
> >
> >Alexandre Tessarollo wrote:
> > >
> > > Marcelo Rufino de Oliveira wrote:
> > >
> > > > Abaixo vão 2 problemas de combinatória/jogos que eu ainda não consegui
> > > > fazer.
> > > > Já mandei estas mesmas duas questões anteriormente para a lista mas
> > > > infelizmente ninguém se manifestou... vamos ver se desta vez alguém
> >pode me
> > > > ajudar.
> > > > Já agradeço, de antemão, aos participantes da lista que tentarem fazer
> >algum
> > > > dos problemas, pois estes não são elementares.
> > > >
> > > > 1) O conjunto {1, 2, ..., 49} é particionado em 3 subconjuntos
> >disjuntos.
> > > > Mostre que ao menos um dos subconjuntos possui três números a, b e c
> >tais
> > > > que a + b = c.
> > > >
> > >
> > > Hum, vamos ver...
> > > 1a hipótese: Separamos de acordo com o resto na divisão por 3.
> > >
> > > Assim, temos o grupo que resta 1, o que resta 2 e o que não resta nada.
> >Neste
> > > último, basta pegar números a=3k, b=3j e c=3(k+j). Naturalmente, k e j
> >são
> > > naturais não-nulos, k é diferente de j e k+j<17. (Isto para que a,b e c
> >estejam
> > > no conjunto original {1,..,49})
> > >
> > > Ih, tô vendo que vai dar um certo trabalho e eu tenho aula daqui a
> >dez
> > > minutos... Bem, veja se consegue mostrar o que o problema pede pensando
> >nessas
> > > possibilidades. Talvez tenha uma maneira mais direta, não sei. Vou ver
> >se até
> > > amanhã eu consigo resolver e digitar tudo.
> > >
> > > []'s
> > >
> > > Alexandre Tessarollo
> > >
> > > PS: Sei que não é a resolução completa, mas de repente ajuda... :-)
> > >
> > > >
> > > > 2) Dado um retângulo 1993x1994, dois jogadores (um de cada vez)
> >escreve os
> > > > números 0 ou 1 nas casas. Quando o tabuleiro  está completo seja A o
> >máximo
> > > > valor das somas das 1993 linhas e B o máximo valor das somas das
> >colunas. No
> > > > caso em que A > B o primeiro ganha, no outro caso B ganha. Quem possui
> >uma
> > > > estratégia vencedora?
> > > >
> > > > Falou,
> > > > Marcelo Rufino
> 
> _
> Get Your Private, Free E-mail from MSN Hotmail at http://www.hotmail.com.



Re: =?x-user-defined?q?Quest=F5es?= de =?x-user-defined?q?combinat=F3ria=2Fjogos?=

2001-06-24 Por tôpico Augusto Morgado

Leia a1 como a indice 1.
Observe inicialmente que a diferença entre dois elementos
distintos(maior-menor) do conjunto é ainda um relemento do conjunto.
Fixe a1=1. Considere as 48 diferenças a2-a1,..., a48-a1.Algum dos três
conjuntos conterá pelo menos dezesseis dessas 48 diferenças. Sejam b1,
b2,...,b16 essas diferenças e seja X o conjunto ao qual elas pertencem. 
Considere as 15 diferenças b2-b1=c1,...,b16-b1=c15. 
Se alguma dessa diferenças pertencer a X, X conterá b1, bk-b1 e bk, isto
é, as-a1, aj-a1-(as-a1)=aj-as e aj-a1; fim, pois o terceiro é a soma dos
dois primeiros.

Caso contrário as 15 diferenças pertencerao aos outros dois conjuntos Y
e Z, havendo em um dos conjuntos, digamos Y, pelo menos 8 dessas
diferenças.Chamemos essas diferenças de d1,...,d8.Considere as 7
diferenças d2-d1,...,d8-d1.Note que essas diferenças sao diferenças
entre bês e portanto diferenças entre elementos da sequencia dos a,
estando ja excluida a possibilidade de alguma delas pertencer a X.
Se alguma dessa diferenças pertencer a Y, Y conterá d1, dp-d1 e dp, isto
é, bm-b1=ar-a1-(as-a1)=ar-as, bn-a1-(bm-b1)=bn-bm=(au-a1)-(ar-a1)=au-ar
e bn-b1=(au-a1)-(as-a1)=au-as; fim, pois o terceiro é a soma dos dois
primeiros.
Caso contrário, as 7 diferenças d2-d1=e1,...,d8-d1=e7 pertencerao a Z.
As seis diferenças e2-e1,...,e7-e1 pertencerao a Z pois sao diferenças
entre termos da sequencia dos d, estando ja excluida a possibilidade de
pertencerem a Y. Entao Z contera e1, ef-e1, ef ...fim, pois o terceiro é
a soma dos dois primeiros.
 


Alexandre Tessarollo wrote:
> 
> Marcelo Rufino de Oliveira wrote:
> 
> > Abaixo vão 2 problemas de combinatória/jogos que eu ainda não consegui
> > fazer.
> > Já mandei estas mesmas duas questões anteriormente para a lista mas
> > infelizmente ninguém se manifestou... vamos ver se desta vez alguém pode me
> > ajudar.
> > Já agradeço, de antemão, aos participantes da lista que tentarem fazer algum
> > dos problemas, pois estes não são elementares.
> >
> > 1) O conjunto {1, 2, ..., 49} é particionado em 3 subconjuntos disjuntos.
> > Mostre que ao menos um dos subconjuntos possui três números a, b e c tais
> > que a + b = c.
> >
> 
> Hum, vamos ver...
> 1a hipótese: Separamos de acordo com o resto na divisão por 3.
> 
> Assim, temos o grupo que resta 1, o que resta 2 e o que não resta nada. Neste
> último, basta pegar números a=3k, b=3j e c=3(k+j). Naturalmente, k e j são
> naturais não-nulos, k é diferente de j e k+j<17. (Isto para que a,b e c estejam
> no conjunto original {1,..,49})
> 
> Ih, tô vendo que vai dar um certo trabalho e eu tenho aula daqui a dez
> minutos... Bem, veja se consegue mostrar o que o problema pede pensando nessas
> possibilidades. Talvez tenha uma maneira mais direta, não sei. Vou ver se até
> amanhã eu consigo resolver e digitar tudo.
> 
> []'s
> 
> Alexandre Tessarollo
> 
> PS: Sei que não é a resolução completa, mas de repente ajuda... :-)
> 
> >
> > 2) Dado um retângulo 1993x1994, dois jogadores (um de cada vez) escreve os
> > números 0 ou 1 nas casas. Quando o tabuleiro  está completo seja A o máximo
> > valor das somas das 1993 linhas e B o máximo valor das somas das colunas. No
> > caso em que A > B o primeiro ganha, no outro caso B ganha. Quem possui uma
> > estratégia vencedora?
> >
> > Falou,
> > Marcelo Rufino



Re: =?x-user-defined?q?d=FAvida?=

2001-06-24 Por tôpico Augusto Morgado

Sistemas equivalentes sao os que tem as mesmas solucoes.
Por exemplo x+y=0, x-y=0 e 3x+4y=0, x+2y=0 sao equivalentes, mas x+y=0,
x-y=0 e 2x+y=0, 4x+2y=0 nao sao.
Morgado

> Eder wrote:
> 
> Saudações a todos da lista,
>  
> Eu gostaria de perguntar a vcs o seguinte: dois sistemas
> homogêneos,mesmo com equações diferentes,podem ser considerados
> equivalentes?
>  
> Obrigado por qualquer comentário.



Re: 0,8... + 0,1... = 1 ?!

2001-06-23 Por tôpico Augusto Morgado

A calculadora tem limitação nas casas decimais que pode exibir; então,
ela exibe os resultados arredondados. Uma calculadora que tivesse
limitaçao a duas casa decimais exibiria
1/3=0,33
2/3=0,67
1/4=0,25
1/100=0,01
1/1000=0,00
Agora, 8/9=0,88(portanto, estava certo) e se a calculadora tem
limitaçao a 10 casas exibira 0,888 888 888 9.
Morgado
Leonardo Motta wrote:
> 
> Isso parece estar errado:
> > 8/9 = 0,888...
> > 1/9 = 0,111...
> 
> Se vc usar uma calculadora científica, verá que o resultado correto é:
> 
> 8/9 = 0,8889
> 1/9 = 0,
> 
> Se você somar os dois números, dá sempre 1. Ou você pode usar uma
> calculadora científica e entrar a expressão equivalente (8/9) + (1/9), e
> verá que ela dá resultado 1. Não tive coragem de verificar se a divisão 8/9
> para onde a calculadora sugere. Usei a calculadora científica do WINDOWS! :)



Re: 0,8... + 0,1... = 1 ?!

2001-06-23 Por tôpico Augusto Morgado

Eu acho estranho que alguns achem estranho 0,999...=1 e não achem
estranho 0,333...=1/3.
Morgado

Gustavo Nunes Martins wrote:
> 
> 8/9 = 0,888...
> 1/9 = 0,111...
> 0,888... + 0,111... = 0,999...
> 8/9 + 1/9 = 9/9 = 1
> Entao:
> 0,999... = 1
> 
> Nao entendo que 0,999... seja IGUAL a 1. Suponho que seja diferente de
> 1. Alguem pode me explicar o que esta coisa significa?
> 
> Atenciosamente,
> Gustavo



Re: Determinante

2001-06-16 Por tôpico Augusto Morgado

Apesar da exposiçao do Frederico Pessoa ter sido magnifica, certamente
ele nao vai conseguir expor aqui na lista toda a teoria dos
determinantes.
Eu tambem passei pelas mesmas dificuldades quando aluno do segundo ano
do cientifico (nome que na minha epoca se dava ao ensino medio). Embora
o livro do Ari Quintela provasse as coisas direitinho, surgiam
dificuldades. Eu so consegui entender tudo lendo o Licoes de Algebra e
Analise, de Bento Jesus Caraça.
E muito claro, prova tudo e esta em Portugues.
Morgado



Re: ainda sobre livros...

2001-06-03 Por tôpico Augusto Morgado

Eu e Wagner fizemos uma mini-resenha (muito objetiva; não era crítica e
sim uma descrição resumida)de todos (exceto apenas um: o de pontos
fixos) os livrinhos da MIR. Livrinhos, isto é, livros da série Lições
Populares de Matemática, que são livros pequenos sobre tópicos.
Foi publicada na RPM, seção Livros. O Josimar, que é organizado,
certamente vai descobrir os números em que foram publicadas as resenhas.
As resenhas podem ser muito úteis pois tendo o nome do livro, os autores
e os temas facilitarão buscas em bibliotecas e Internet.   
Morgado
Anderson wrote:
> 
> Os livros da editora Mir sao varios... pelo que eu vi no site da editora
> atual (nao lembro o site mas procura no www.google.com que vc encontra) eles
> reeditaram alguns dos livros desta antiga editora... eu tenho 4 livros dela
> ambos muito bons...
> Problemas Selecionados de Fisica Elementar - Saraeva
> Elementos de Matematicas superiores - I. L. Zaitsev (Espanhol)
> Calculo Diferencial e Integral I e II - N. Piskunov
> 
> Anbraços,
> Anderson
> 
> - Original Message -
> From: Marcelo Souza <[EMAIL PROTECTED]>
> To: <[EMAIL PROTECTED]>
> Sent: Saturday, May 26, 2001 3:50 PM
> Subject: Re: ainda sobre livros...
> 
> > que ignorancia nada, fala, é oi marcelo de novo. É o seguinte, alguns
> livros
> > da MIR de geometria são muito cotados, livros de problemas em geral tb o
> que
> > é o forte da MIR), msas estes livros nós so achamos em sebos. Vale a pena
> > dar uma  vasculhada. Os titulos eu naum sei de cabeça, mas xa vê!! Eu acho
> > que um é Problems in Geometry, sei lah, nem lembro...alguém pode lembrar?
> > abraços
> > Marcelo



Re: =?x-user-defined?q?Infla=E7=E3o?=

2001-05-14 Por tôpico Augusto Morgado

Corrigindo:
O livro chama-se Progressões e Matemática Financeira, editado pela SBM e
os autores são Wagner, Sheila Zani e eu (apesar disso, o livro é bom, eu
garanto).
Morgado


Carlos Frederico Borges Palmeira wrote:
> 
> On Mon, 14 May 2001, Rebeca Tenney wrote:
> 
> >   Eu tava fazendo uma prova da Puc-Rio de alguns anos atrás,
> > e percebi q n tenho a mínima idéia de como se fazer
> > problemas c inflação, ou inflação acumulada, nada disso!! Um
> > absurdo!!
> > Olha só a questao em q eu empaquei:
> >Suponha uma inflãção mensal de 4% durante um ano. De
> > quanto será a inflação acumulada neste ano?
> >
> >  Muito obrigada pela paciência,
> >*Beca*
> > 
> > Don't E-Mail, ZipMail! http://www.zipmail.com/
> >
> inflacao e' progressao geometrica. Em vez de somar 4% a cada mes, a gente
> multiplica por 1,04, isto e', o que custava 100 passa a custar
> 100*1,04=104, que e' o mesmo que dizer 100+4=104. So' que multiplicando
> fica mais facil fazer varios meses de uma vez: em 12 meses da 100 *
> 1,04^12. Ha um livro de progressoes e combinatoria (acho) publicado pela
> SBM do Morgado, Pitombeira e se nao me engano Paulo Cezar Pinto, que
> explica bem essa matematica financeira elementar, tao util para o dia a
> dia, como: preco a vista 100; ou 60 agora e mais 60 daqui a um mes, o que
> da 120, logo o juro e' de 20% ao mes. Isso e' o que diz o vendedor, na
> verdade o juro e' de 50%, ja que 60 de entrada voce ja paga de qualquer
> jeito. os 40 restantes e' que estao virando 60 em um mes.!
> 
> Fred



Re: Woods

2001-04-03 Por tôpico Augusto Morgado

Certamente o livro a que me referi e que foi publicado em portugues era
outro.
Morgado
Leonardo Motta wrote:
> 
> > Não era
> > um livro de Cálculo Avançado, era um livro de Cálculo.
> 
> Eu encontrei a ref. do livro:
> 
> Woods, Frederick S. "Advanced Calculus: A Course Arranged with Special
> Reference to the Needs of Students of Applied Mathematics". Boston, MA: Ginn
> (1926).
> 
> Ele provavelmente foi lancado no Brasil com o mesmo titulo pq a Editora da
> UnB traduziu como "Calculo Avancado"... Talvez o F. S. Woods tenha publicado
> mais do que um livro de calculo. Pelo titulo da' p/ perceber q esse parece
> ser uma edicao especial voltada para aplicacoes.
> 
> > Certamente você o
> > encontrará em bibliotecas de escolas de engenharia antigas.
> 
> Ou seja, bem longe de onde estou :((



Re: Woods

2001-04-02 Por tôpico Augusto Morgado

Os primeiros livros de Cálculo que foram traduzidos para o português e
editados no Brasil
foram o Granville (na realidade eram 2 ou 3 autores, mas ficou conhecido
como Granville) e o Woods e Bailey. Não tenho o Woods e Bailey; já não
era editado quando entrei para a universidade (tenho 57 anos). Não era
um livro de Cálculo Avançado, era um livro de Cálculo. Certamente você o
encontrará em bibliotecas de escolas de engenharia antigas. 
Leonardo Motta wrote:
> 
> Alow :)
> 
> Sera' q alguem da' lista sabe ou jah ouviu falar do livro "Calculo Avancado"
> ("Advanced Calculus") de Woods (?)? Eu nao consigo achar referencias sobre
> esse livro, exceto q ele foi citado pelo R.P. Feynman nas memorias dele.
> Feynman fala muito bem desse livro, diz que fora um diferencial p/ ele.
> Sera' q alguem conhece alguma edicao do livro?



Re: =?x-user-defined?q?M=FAltiplos?= de 3

2001-03-26 Por tôpico Augusto Morgado

1) a^3-a=a(a^2-1)=a(a-1)(a+1). Em três inteiros consecutivos um é
múltiplo de 3.
2) a^3-b^3 = (a-b)(a^2+ab+b^2). 
a)Se a-b é múltiplo de 3, a^3-b^3 também será.
b) Vamos fazer uma tabela dos restos possíveis na divisão por 3.
a b  a-ba^3   b^3  a^3-b^3 
0  0  0 00 0
0  1  2 01 2
0  2  1 02 1
1  0  1 10 1
1  1  0 11 0
1  2  2 12 2
2  0  2 20 2
2  1  1 21 1
2  2  0 22 0  
Se a^3-b^3 é múltiplo de 3 (linhas 1, 5, 9) , a-b também é.

> Rubens wrote:
> 
> Uma ajuda:
>  
> 1)Mostre que a^3 - a é múltiplo de 3, para todo a inteiro.
>  
> 2) Mostre quer a^3 - b^3 é múltiplo de 3 se, e somente se, a-b é
> múltiplo de 3.
>  
> Obrigado
>  
>  



Re: =?x-user-defined?q?Demonstra=E7=E3o?=

2001-03-23 Por tôpico Augusto Morgado

a^2 + b^2 + c^2 = a.b + a.c + b.c 
2a^2 +2 b^2 + 2c^2 -2 a.b -2 a.c -2 b.c = 0
(a~b)^2 + (b-c)^2+ (c-a)^2 = 0
a=b=c
"João Paulo Paterniani da Silva" wrote:
> 
>Olá.
>Mostre que se a^2 + b^2 + c^2 = a.b + a.c + b.c , então a=b=c.
> 
> João Paulo Paterniani da Silva
> 
> _
> Get Your Private, Free E-mail from MSN Hotmail at http://www.hotmail.com.



Re: Problemas com 111...'s

2001-03-18 Por tôpico Augusto Morgado

1)Todos eles dao resto 3 na divisao por 4.
Basta ver que isso eh verdade para o 11 e que todos os demais sao a soma
de 11 com um multiplo de 100 (o qual evidentemente eh multiplo de 4).
Os quadrados dos pares dao resto 0 na divisao por 4 e os dos impares dao
resto 1.

> Rubens wrote:
> 
> Prezados Amigos, solicito colaboração com os problemas abaixo:
>  
> 1) Demonstrar que nenhum inteiro da sequência:
>  11, 111, , 1, ...
>    é um quadrado perfeito.
>  
> 2) Mostrar que o inteiro  111...11  com n algarismos 1 é composto se n
> é composto.
>  
> 3) Mostrar que os os inteiros
>     , 11, , , ...11, ...
>    cada um dos quais é formado por um número par de algarismos 1, são
> todos   compostos.
>  
> 4) Calcular a soma: 1 + 11 + 111 + ... + 111...1
>     onde a última parcela é formada de n algarismos 1.
>  
> Obrigado a todos.
> Rubens



Re: Problemas com 111...'s

2001-03-18 Por tôpico Augusto Morgado

3) Todos sao divisiveis por 11. Isso se pode ver pelo classico criterio
de divisibilidade por 11 ou com argumentos do tipo 11 = 11 +11*10^2+
11*10^4.
2) 11...1= 1+1*10+1*10^2+...+1*10^(n-1)= (10^n - 1)/9. Se n eh
composto, n=ab com a e b inteiros e diferentes de 1. 
10^(ab)-1 é divisivel por 10^a-1 (X), isto eh, 
10^(ab)-1 = (10^a-1)* P, P inteiro.
Portanto, ...1= P*[(10^a-1)/9].
Como (10^a-1)/9 eh inteiro(X), isso prova que 
...1 eh composto.
As passagens marcadas com X sao justificadas pela identidade x^n-1 =
(x-1)*[(x^(n-1))+...+x^2+x+1]


 Rubens wrote:
> 
> Prezados Amigos, solicito colaboração com os problemas abaixo:
>  
> 1) Demonstrar que nenhum inteiro da sequência:
>  11, 111, , 1, ...
>    é um quadrado perfeito.
>  
> 2) Mostrar que o inteiro  111...11  com n algarismos 1 é composto se n
> é composto.
>  
> 3) Mostrar que os os inteiros
>     , 11, , , ...11, ...
>    cada um dos quais é formado por um número par de algarismos 1, são
> todos   compostos.
>  
> 4) Calcular a soma: 1 + 11 + 111 + ... + 111...1
>     onde a última parcela é formada de n algarismos 1.
>  
> Obrigado a todos.
> Rubens



Re: Ainda sobre o quadrado

2001-03-12 Por tôpico Augusto Morgado

Porque a soma de todos os numeros do quadrado eh a soma dos números 1,
2, ..., n^2, que eh igual a n^2(n^2+1)/2. dividida em n colunas de igual
soma, a soma de cada coluna serah n(n^2+1)/2.

[EMAIL PROTECTED] wrote:
> 
> Mais uma coisinha:
> Por que a soma de um quadrado mágico (de ordem n) é : n(1+n^2)/2?
> Qual a demonstraçao?
> Abraços
> 
> MailBR - O e-mail do Brasil -- http://www.mailbr.com.br
> Faça já o seu. É gratuito!!!



Re: somatorio

2001-03-12 Por tôpico Augusto Morgado

Corrigindo:
Consultem funçao psi, funçao digama e constante de Euler (ou constante
de Euler-Mascheroni).

Augusto Morgado wrote:
> 
> Acho que todos estao respondendo o que nao foi perguntado. Perguntou-se
> quanto valia o somatório de 1 a n e nao de 1 a infinito.
> Se o n eh grande o somatorio eh bem aproximado por logaritmo natural de
> n.
> Nao ha modo "elementar" de calcular o somatorio. Entretanto ele pode ser
> expresso em termos de funçoes tabeladas (no seculo passado; hoje seria
> melhor dizer mapleadas). Consultem, em um livro de Calculo, funçao
> digama ou funçao psi.
> 
> Fábio Arruda de Lima wrote:
> >
> > Oi amigo,
> > Inicialmente, seria interessante você adquirir o livro do Prof. Elon Lages
> > Lima, Curso de Análise, e dar uma lida no Capítulo de Seqüências e Séries de
> > números reais.
> > Entretanto, como esclarecimento. Trago o seguinte Teorema:
> > "Se Somatório de An é uma série convergente então o limite An = 0."
> > Entretanto a recíproca não é verdadeira e o contra-exemplo clássico é
> > exatemente somatório de 1/n. Esta série diverge!
> > Gostaria de complementar o assunto trazendo uma pequena técnica (aprendi
> > vendo em muitos livros) para o calculo de somatório.
> > Busque transformar o somatorio do termo geral em diferença de dois termos.
> > Por exemplo:
> > Somatório (1/(n)(n+1) = A/n - B/(n+1) = (An + A -Bn)/ (n)(n+1)
> > A-B=0
> > A=1
> > Portanto, B=1.
> > Assim, temos Somatório 1/(n)(n+1) = 1/n - 1/(n+1)
> > Temos: 1- 1/2
> > 1/2 - 1/3
> > 1/3 - 1/4
> > .
> > 1/n - 1/(n+1)
> > Simplificando positivos e negativos, temos:
> > Soma = 1 - 1/(n+1)
> >
> > - Original Message -
> > From: <[EMAIL PROTECTED]>
> > To: <[EMAIL PROTECTED]>
> > Sent: Saturday, March 10, 2001 5:00 PM
> > Subject: somatorio
> >
> > > Podem me ajudar com este somatorio?
> > >
> > > 1/k;com K variando de 1 ate n
> > >
> > >
> > >



Re: =?x-user-defined?q?Defini=E7=E3o?= de Ln, conflitos internos eNavalha de Occam

2001-03-12 Por tôpico Augusto Morgado

Isso eh resultado de questoes de vestibular serem feitas por pedagogos
que adoram coisas como "mais certo", "mais honesto", "mais gravida",
etc.
Se as alternativas sao iguais, tipo 2 e 4/2, se uma estiver correta a
outra tambem estarah.
Onde ja se viu mais certo!
Morgado
Benjamin Hinrichs wrote:
> 
> Eduardo Wagner wrote:
> 
> > Neste ponto nao se preocupe. Nao vai acontecer. 1/raiz(3) eh igual
> > a raiz(3)/3 e nao pode haver duas opcoes certas em uma questao.
> 
> Minha fonte confiável fez vestibular há uns vários anos atrás (mais que
> eu de vida tenho) e relata que em uma questão havia quatro opçõs
> equivalentes, duas a duas (uma era o número simplificado da outra). Foi
> simples, marcou a que não era proporcional a nenhuma outra e errou;
> explicação: quando duas opções certas são proporcionais, a correta é a
> mais simplificada. Claro, "entia non sunt multiplicanda praeter
> necessitatem". Não sei se isso valeria também para o caso das raízes,
> espero que não caia no próximo vestibular.
> 
> Grande abraço,
> Benjamin Hinrichs



Re: somatorio

2001-03-12 Por tôpico Augusto Morgado

Acho que todos estao respondendo o que nao foi perguntado. Perguntou-se
quanto valia o somatório de 1 a n e nao de 1 a infinito.
Se o n eh grande o somatorio eh bem aproximado por logaritmo natural de
n.
Nao ha modo "elementar" de calcular o somatorio. Entretanto ele pode ser
expresso em termos de funçoes tabeladas (no seculo passado; hoje seria
melhor dizer mapleadas). Consultem, em um livro de Calculo, funçao
digama ou funçao psi.

Fábio Arruda de Lima wrote:
> 
> Oi amigo,
> Inicialmente, seria interessante você adquirir o livro do Prof. Elon Lages
> Lima, Curso de Análise, e dar uma lida no Capítulo de Seqüências e Séries de
> números reais.
> Entretanto, como esclarecimento. Trago o seguinte Teorema:
> "Se Somatório de An é uma série convergente então o limite An = 0."
> Entretanto a recíproca não é verdadeira e o contra-exemplo clássico é
> exatemente somatório de 1/n. Esta série diverge!
> Gostaria de complementar o assunto trazendo uma pequena técnica (aprendi
> vendo em muitos livros) para o calculo de somatório.
> Busque transformar o somatorio do termo geral em diferença de dois termos.
> Por exemplo:
> Somatório (1/(n)(n+1) = A/n - B/(n+1) = (An + A -Bn)/ (n)(n+1)
> A-B=0
> A=1
> Portanto, B=1.
> Assim, temos Somatório 1/(n)(n+1) = 1/n - 1/(n+1)
> Temos: 1- 1/2
> 1/2 - 1/3
> 1/3 - 1/4
> .
> 1/n - 1/(n+1)
> Simplificando positivos e negativos, temos:
> Soma = 1 - 1/(n+1)
> 
> - Original Message -
> From: <[EMAIL PROTECTED]>
> To: <[EMAIL PROTECTED]>
> Sent: Saturday, March 10, 2001 5:00 PM
> Subject: somatorio
> 
> > Podem me ajudar com este somatorio?
> >
> > 1/k;com K variando de 1 ate n
> >
> >
> >



Re: ime 2001

2001-03-04 Por tôpico Augusto Morgado

To maluco!
Consegui nao ler a segunda soluçao.
Desculpem.
Morgado

Augusto Morgado wrote:
> 
> Alem dessas (otimas) soluçoes, para mostrar a divisibilidade por 5
> poder-se-ia usar (eis o canhao matando uma mosca) o Pequeno Teorema de
> Fermat.
> 
> Paulo Santa Rita wrote:
> >
> > Ola Falows e Amigos da Lista,
> >
> > Se o algarismo das unidades de "K^5" e de "K" sao
> > iguais, entao a diferenca "K^5 - K" termina em
> > zero, vale dizer : ela e multiplo de dez.
> >
> > Claramente que "K^5 - K" e multiplo de dois,
> > qualquer que seja o natural "K". Para ver isso,
> > note que se supormos que "K" e par, entao "K^5"
> > sera necessariamente par e, portanto, a diferenca
> > "K^5 - K" sera do tipo "par - par" que e par; por
> > outro lado, supondo "K" impar, "K^5" sera impar e
> > ,neste caso, diferenca "K^5 - K" sera do tipo
> > "impar - impar" que e par.
> >
> > Resta provarmos que "K^5 - K" e tambem multiplo
> > de cinco. Existe uma grande quantidade de formas
> > de se fazer isso ...
> >
> > 1 FORMA ( Estilo "Aluno de 8 serie ) :
> > Se "K" for multiplo de cinco, entao
> > o fato de "K^5 - K" poder ser colocado na forma
> > K(K^4 - 1) mostra que esta diferenca tambem e
> > multiplo de cinco. Se "K" nao for multiplo de
> > cinco, entao, pelo algoritmo da divisao, ele
> > podera ser colocado na forma 5*q + r, com 0 < r < 5.
> >
> > Como :
> >
> > K^5 - K = K(K^4 - 1) = K(K^2 - 1)(K^2 + 1)
> > K^5 - K = K(K - 1)(K + 1)(K^2 + 1)
> >
> > Se r=1, "K^5 - K" se transformara em
> > (5p+1)*5p*(5p+2)[(5p+1)^2 + 1] ...
> > Multiplo de 5 !
> >
> > Se r=4, "K^5 - K" se transformara em
> > (5p+4)*(5p+3)*(5p+5)*[(5p+4)^2 + 1]
> > 5*(5p+4)*(5p+3)*(p+1)*[(5p+4)^2 + 1] ...
> > Multiplo de 5 !
> >
> > Se r=2 ou r=3 o fator "K^2 + 1" ira se
> > transformar, respectivamente, em
> > (25p^2 + 20p + 5) =5*(5p^2 + 4p +1) e
> > (25p^2 + 30p + 10)=5*(5p^2 + 6p + 2),
> > ambos multiplos de 5 !
> >
> > Esgotadas as hipoteses possiveis sobre r,
> > so nos resta admitir que "K^5 - K" e sempre
> > multiplo de 5.
> >
> > 2 FORMA ( Estilo "Sintetico - Como eu faria" ) :
> >
> > Vemos que K^5 - K = K(K^4 - 1). Se K for
> > multiplo de 5, claramente que K(K^4 - 1)
> > tambem sera. Se nao for, entao, pelo teorema
> > de Fermat, K^4 e congruo a 1 modulo 5 e,
> > portanto, K^4 e 1 deixam o mesmo resto quando
> > dividos por 5 e, assim, k^4 - 1 e multiplo de 5.
> >
> > 3 FORMA ( Estilo "Rolo compressor" )
> >
> > E como voce fez, listando, pelo que entendi,
> > todas as possibilidades de combinacoes. E valido.
> >
> > Nota : Existe um teorema ( das quatro cores )
> > cuja primeira prova consistiu em exibir todas
> > as combinacoes possiveis ... Muitas pessoas nao
> > aceitam tal prova, outras aceitam ... Em minha
> > opiniao ( fraca, em face do que podem dizer os
> > Grandes Professores que orientam Nossa Lista ),
> > uma "Prova por Enumeracao" e um indicativo da
> > falta de algum(ns) conceito(s) do(s) qual(is)
> > o fato provado por enumeracao possa ser derivado
> > como consequencia logica !
> >
> > 4 FORMA ( Estilo "indutor" )
> >
> > A praxis seria supor que "P^5 - P" (K=P) e
> > divisivel por 5 e mostrar que para (K=P+1)
> > obrigatoriamente tambem seria. Nao vou fazer,
> > mas tenho certeza que e tao simples como todos
> > os outros casos ...
> >
> > Eu acho que esta bom, mas neste momento estou
> > vendo duas outras formas diferentes de chegar
> > a este resultado ... Isto mostra a simplicidade
> > da questao e a riqueza das tecnicas matematicas.
> >
> > Um Grande Abraco pra voce
> > Um Grande abraco pra todos os colegas da lista
> >
> > Paulo Santa Rita
> > 6,1158,02032001
> >
> > Nota : Existe um Teorema ( das quatro cores ) em que a prova
> >
> >  
> >
> >  
> >
> > >From: "Exercicio~®"
> > >Reply-To: [EMAIL PROTECTED]
> > >To: [EMAIL PROTECTED]
> > >Subject: ime 2001
> > >Date: Fri, 02 Mar 2001 00:27:53 -0300
> > >
> > >
> > >
> > > Olá pessoal!
> > >
> > > Essa questão foi do último vestibular do ime. Alguém poderia
> > apresentar

Re: ime 2001

2001-03-02 Por tôpico Augusto Morgado

Alem dessas (otimas) soluçoes, para mostrar a divisibilidade por 5
poder-se-ia usar (eis o canhao matando uma mosca) o Pequeno Teorema de
Fermat. 

Paulo Santa Rita wrote:
> 
> Ola Falows e Amigos da Lista,
> 
> Se o algarismo das unidades de "K^5" e de "K" sao
> iguais, entao a diferenca "K^5 - K" termina em
> zero, vale dizer : ela e multiplo de dez.
> 
> Claramente que "K^5 - K" e multiplo de dois,
> qualquer que seja o natural "K". Para ver isso,
> note que se supormos que "K" e par, entao "K^5"
> sera necessariamente par e, portanto, a diferenca
> "K^5 - K" sera do tipo "par - par" que e par; por
> outro lado, supondo "K" impar, "K^5" sera impar e
> ,neste caso, diferenca "K^5 - K" sera do tipo
> "impar - impar" que e par.
> 
> Resta provarmos que "K^5 - K" e tambem multiplo
> de cinco. Existe uma grande quantidade de formas
> de se fazer isso ...
> 
> 1 FORMA ( Estilo "Aluno de 8 serie ) :
> Se "K" for multiplo de cinco, entao
> o fato de "K^5 - K" poder ser colocado na forma
> K(K^4 - 1) mostra que esta diferenca tambem e
> multiplo de cinco. Se "K" nao for multiplo de
> cinco, entao, pelo algoritmo da divisao, ele
> podera ser colocado na forma 5*q + r, com 0 < r < 5.
> 
> Como :
> 
> K^5 - K = K(K^4 - 1) = K(K^2 - 1)(K^2 + 1)
> K^5 - K = K(K - 1)(K + 1)(K^2 + 1)
> 
> Se r=1, "K^5 - K" se transformara em
> (5p+1)*5p*(5p+2)[(5p+1)^2 + 1] ...
> Multiplo de 5 !
> 
> Se r=4, "K^5 - K" se transformara em
> (5p+4)*(5p+3)*(5p+5)*[(5p+4)^2 + 1]
> 5*(5p+4)*(5p+3)*(p+1)*[(5p+4)^2 + 1] ...
> Multiplo de 5 !
> 
> Se r=2 ou r=3 o fator "K^2 + 1" ira se
> transformar, respectivamente, em
> (25p^2 + 20p + 5) =5*(5p^2 + 4p +1) e
> (25p^2 + 30p + 10)=5*(5p^2 + 6p + 2),
> ambos multiplos de 5 !
> 
> Esgotadas as hipoteses possiveis sobre r,
> so nos resta admitir que "K^5 - K" e sempre
> multiplo de 5.
> 
> 2 FORMA ( Estilo "Sintetico - Como eu faria" ) :
> 
> Vemos que K^5 - K = K(K^4 - 1). Se K for
> multiplo de 5, claramente que K(K^4 - 1)
> tambem sera. Se nao for, entao, pelo teorema
> de Fermat, K^4 e congruo a 1 modulo 5 e,
> portanto, K^4 e 1 deixam o mesmo resto quando
> dividos por 5 e, assim, k^4 - 1 e multiplo de 5.
> 
> 3 FORMA ( Estilo "Rolo compressor" )
> 
> E como voce fez, listando, pelo que entendi,
> todas as possibilidades de combinacoes. E valido.
> 
> Nota : Existe um teorema ( das quatro cores )
> cuja primeira prova consistiu em exibir todas
> as combinacoes possiveis ... Muitas pessoas nao
> aceitam tal prova, outras aceitam ... Em minha
> opiniao ( fraca, em face do que podem dizer os
> Grandes Professores que orientam Nossa Lista ),
> uma "Prova por Enumeracao" e um indicativo da
> falta de algum(ns) conceito(s) do(s) qual(is)
> o fato provado por enumeracao possa ser derivado
> como consequencia logica !
> 
> 4 FORMA ( Estilo "indutor" )
> 
> A praxis seria supor que "P^5 - P" (K=P) e
> divisivel por 5 e mostrar que para (K=P+1)
> obrigatoriamente tambem seria. Nao vou fazer,
> mas tenho certeza que e tao simples como todos
> os outros casos ...
> 
> Eu acho que esta bom, mas neste momento estou
> vendo duas outras formas diferentes de chegar
> a este resultado ... Isto mostra a simplicidade
> da questao e a riqueza das tecnicas matematicas.
> 
> Um Grande Abraco pra voce
> Um Grande abraco pra todos os colegas da lista
> 
> Paulo Santa Rita
> 6,1158,02032001
> 
> Nota : Existe um Teorema ( das quatro cores ) em que a prova
> 
>  
> 
>  
> 
> >From: "Exercicio~®"
> >Reply-To: [EMAIL PROTECTED]
> >To: [EMAIL PROTECTED]
> >Subject: ime 2001
> >Date: Fri, 02 Mar 2001 00:27:53 -0300
> >
> >
> >
> > Olá pessoal!
> >
> > Essa questão foi do último vestibular do ime. Alguém poderia
> apresentar
> >uma resolução formal para essa questão?
> >
> >
> > ( IME - 2001 )
> >
> > Prove que para qualquer número inteiro K, os números K e K^5
> terminam
> >sempre com o mesmo algarismo ( algarismo das unidades).
> >
> > Eu faria assim:
> >
> >K = R_n onde n varia de 0 a 9 e R é qq número inteiro.
> >
> >K = R_0
> >K^5 = R_0 × R_0 × R_0 × R_0 × R_0 = T_0, onde T é qq número inteiro
> >
> > K = R_1
> > K^5 = R_1 ×R_1 ×R_1 ×R_1 ×R_1 = T_1, onde T é qq número inteiro.
> >
> > K = R_2
> > K^5 = R_2 ×R_2 ×R_2 ×R_2 ×R_2 = T_2, onde T é qq número inteiro.
> >.
> >.
> >.
> >.
> >.
> >.
> > K = R_9
> > K^5 = R_9 ×R_9 ×R_9 ×R_9 ×R_9 = T_9, onde T é qq número inteiro.
> >
> >
> > Agora fica a minha dúvida: Se num problema de demostraçao, caso eu
> >consiga expor para o examinador TODOS os casos existentes(desde q
> seja
> >viável, como nesse problema) para tal demostraçao, eu preciso
> >necessariamente utilizar variáveis literais?
> >
> > No caso, se eu estivesse fazendo essa prova, eu escreveria de R_0
> até
> >R_9, integralmente, ou seja, nao existiria as reticencias q eu
> coloquei
> >entre R_2 e R_9 para poupar um pco + meu tempo..
> >
> > Obrigado.
> >
> >
> > Falow's
> >
> > Exercicio~®
> >
> > http://members.nbci.com/exercicio
> > ICQ # 102856897
> >
> >
> >
> >
> 
> 

Re: problema dos baldes

2001-03-01 Por tôpico Augusto Morgado

A RPM, nao sei em que numero, publicou um artigo do Pitombeira sobre
esses problemas.
Morgado

Luis Lopes wrote:
> 
> Sauda,c~oes,
> 
> O problema
> "Como 'e poss'ivel retirar do mar exatamente 6 litros de 'agua tendo apenas
> dois recipientes, um de 4 e outro de 9 litros?"
> 'e bem conhecido: apareceu numa Superinteressante e at'e num filme com o
> Bruce Willis (mas em ambos talvez n~ao com os mesmos dados).
> 
> Seja o par (i,j), em que i representa a quantidade de litros no recipiente
> de 4 litros, e j no de 9. Uma poss'ivel solu,c~ao, com uma seq"u^encia de
> enchimentos e esvaziamentos dos recipientes seria: (0,9); (4,5); (0,5);
> (4,1); (0,1); (1,0); (1,9); (4,6); (0,6).
> 
> Podemos colocar algumas perguntas (para este caso particular e um geral ):
> 
> 1) Exist^encia. O problema tem sempre solu,c~ao? Que condi,c~oes devemos
> impor a i e j para que o problema seja poss'ivel?
> 
> 2) Unicidade. O problema tem solu,c~ao 'unica? Como garantir isso?
> 
> 3) Algoritmo. Como gerar todas as solu,c~oes? Qual seria a mais r'apida
> (menos movimentos), como no filme?
> 
> [ ]'s
> Lu'is



Re: Limites

2001-02-02 Por tôpico Augusto Morgado

Porque a resposta do livro estah errada.
Agora a pergunta interessante seria: qual o valor do limite se no
denominador estivesse 9 no lugar do 3.

> wfs007 wrote:
> 
> Ola pessoal. Estou enviando este e-mail porque me deparei com a
> seguinte questao:
>  
> Determine o limite da funcao abaixo:
> y=(x+3)/[(x^2)-3] para x tendendo para -3
>  
> Como estou estudando por conta propria nao sei o por que de a resposta
> do gabarito ser -1/6. Em minhas contas aparece 0 (zero). O que esta
> errado no meu raciocinio.
>  
> Obrigado.
>  
> PS.: vcs enviar a resolucao junto para saber como eh. E tambem podem
> explicar um metodo mais simples do que sair substituindo x por -3,1;
> -3,01; -2,9; -2,99. Isso eh, se existir tal metodo.
>  
> So nao entendo porque numa outra questao: y=[(x^2)+1)/x+1 para x
> tendendo a 2, simplesmente substitui o x da funcao por 2 e encontrei o
> resultado do gabarito (5/3). Vcs poderiam me explicar melhor isso.
>  
> Talvez ate mesmo algum site.
>  
> Valeu.



Re: polinomial

2001-01-20 Por tôpico Augusto Morgado

Vou fazer um comentário idiota, mas tenho visto tanta bobagem a esse
respeito em vestibulares (UNIRIO, UFF, etc...)...que penso valer a pena
realçar isso.
 Tudo isso diz respeito a polinomios de coeficientes reais. O polinomio
x-i, por exemplo,
eh de grau impar e nao possui nenhuma raiz real.

Rogerio Fajardo wrote:
> 
> Uma equação polinomial de grau n tem n raízes (distintas ou não, reais
> ou não). Acontece que todas as raízes complexas vêm aos pares, pois se
> a+bi é uma raiz de uma equação polinomial, seu conjugado a-bi também é
> uma raiz dessa equação. Logo, uma equação polinomial de grau ímpar tem
> pelo menos uma raiz real, pois tem número ímpar de raízes.
> 
> Se vc pensar em no gráfico da função polinomial, fica imediato que uma
> função de grau ímpar cruza o eixo x pelo menos uma vez, pois se ela
> começa crescendo desde o menos infinito, ela terminará crescendo até o
> mais infinito, e vice-versa, sendo obrigatória a passagem pelo zero.
> 
> Rogério 
> 
> >From: "Henrique Lima Santana"
> >Reply-To: [EMAIL PROTECTED]
> >To: [EMAIL PROTECTED]
> >Subject: polinomial
> >Date: Sat, 20 Jan 2001 02:40:06 -0200
> >
> >
> >
> > Olá pessoal,
> > Tenho uma dúvida: por quê toda equação polinomial de grau ímpar
> >tem pelo menos uma raiz real?
> > []s, Henrique
> >
> >_
> >Get Your Private, Free E-mail from MSN Hotmail at
> >http://www.hotmail.com.
> >
> 
> --
> Get Your Private, Free E-mail from MSN Hotmail at
> http://www.hotmail.com.



Re: Logaritmos

2000-12-21 Por tôpico Augusto Morgado

As parcelas do segundo parenteses sao os inversos das parcelas do
primeiro.
Portanto o seu problema eh mostrar que (P+Q+R)(1/P+1/Q+1/R) eh maior ou
igual a 9 para P,Q,R positivos. Pela desigualdade das medias a
aritmetica eh maior ou igual a harmonica, isto eh,
(P+Q+R)/3  maior ou igual a 3/(1/P+1/Q+1/R).

> Davidson Estanislau wrote:
> 
> Caros colegas, estou com dificuldades nesta seguinte quest?o:
>  
> Prove que se a,x,y,z s?o n?meros reais maiores que 1 ent?o:  
>  
> Davidson



Re: Conceitos

2000-12-18 Por tôpico Augusto Morgado

Qual eh a editora?

Victor Ferreira Soares wrote:
> 
> O livro é meio infantil mas é muito interessante.
> Sugiro, para os mais curiosos e ávidos, a leitura dos
> livros da série "Teoria do ..." de Isaac Asimov.
> Principalmente o "Teoria dos Números". Lá tem a
> história de boa parte das coisas e é um livro
> ultra-simples e curtinho. Mostra de onde surgiu a
> palavra cálculo e essas definições mais específicas
> também.
> 
> Inté+
> 
> Vik
> 
> =
> "Meu Deus, protegei-me de meus amigos!
> Dos meus inimigos eu me encarregarei."
> 
>   Voltaire
> 
> __
> Do You Yahoo!?
> Yahoo! Shopping - Thousands of Stores. Millions of Products.
> http://shopping.yahoo.com/



Re: ICQ (que tal uma obm-l instantanea?) e series infinitas

2000-12-16 Por tôpico Augusto Morgado

Nao. O problema eh complicado. Depende de um negocio chamado
convergencia uniforme.
As funçoes f(x)=x^n (n natural) sao continuas em [0,1] e o limite quando
n tende a infinito eh uma funçao que vale 0 no intervalo inteiro exceto
no 1, onde vale 1.
Procure um livro de Calculo Avançado tipó Fulks ou melhor ainda (pelo
menos na minha opiniao e na do Jose Paulo) o livro do Brand. 

> Jorge Peixoto Morais wrote:
> 
> * Uma serie infinita de funcoes continuas eh continua? Se voceh acha
> que eh obvio, lembre-se de que todo irracional eh a soma de infinitas
> fracoes. Por isso, tenho essa duvida.
> * Voces usam ICQ? Por que nao nos comunicamos por ICQ, fazendo uma
> obm-l instantanea? Seria como passar das cartas para o telefone. Se
> usam, me avisem, para que criemos uma "comunidade nerd".
> *Toda funcao pode ser entendida como uma serie, como as de Mclaurin?
> Certamente, isso eh verdadeiro para todo polinomio (permite-se que
> termos da seire sejam nulos), para seno, cosseno e tangente, para a
> exp(x), para ln(x)... Vale para toda funcao continua?



Re: =?x-user-defined?q?Algu=E9m?= pode me ajudar?

2000-12-16 Por tôpico Augusto Morgado

8=2/3.log(E}E0)
12=log(E/E0)
E/E0= 10^12
E=7*10^-3*10^12=7*10^5= 700 000kWh

Quando I aumenta 1, o log aumenta 3/2=1,5, ou seja, E fica multiplicado
por 10^1,5=31,6.


> Davidson Estanislau wrote:
> 
> A intensidade I de um terremoto ,medida na escala Richter, ? um
> n?mero que varia de I = 0
> at? I=8,9 para o maior terremoto conhecido .I ? dado pela f?rmula:
>  
> I=2/3=log(E/E0)
> onde E ? a energia liberada no terremoto em quilowatt-hora e
> E0=7*10^-3 kWh
>  
> a)Qual a energia liberada no terremoto de intensidade 8 na escala
> Richter?
> b)Aumentando de uma unidade a intensidade do terremoto por quanto fica
> multiplicada a energia liberada?



Re: =?x-user-defined?q?Logar=EDtmos?=

2000-12-16 Por tôpico Augusto Morgado

2000=2/(2+15*4^(-2x))equivale a 
2+15*4^(-2x)=10
15*4^(-2x)=8
4^(-2x)=8/15
log[4^(-2x)]=log[8/15]
-2xlog4=log8-log15
log4=2log2=0,60
log8=3log2=0,90
log15=log3+log5=log3+log10-log2=1,18
-1,2x=-0,28
x=0,23
> Davidson Estanislau wrote:
> 
> Segundo uma pesquisa, ap?s x meses da constata??o da exixt?ncia
>  de uma epidemia o n? de pessoas por ela atingida ?:
> f(x)=(2/(2+15*4^(-2*x)))
> Segundo log 2= 0,30 e log 3= 0.48 ,daqui a quanto tempo
> aproximadamente o n?mero de pessoas atingidas por essa
>  epidemia ser? de 2000?



Re: Trigonometria-Ajuda Urgente

2000-12-13 Por tôpico Augusto Morgado

Multiplique por sen(pi/65). Use varias vezes que sena.cosa=(sen2a)/2.
Lembre-se ao final que pi/65 e 64pi/65 sao suplementares e tem o mesmo
seno.

> Hugo Iver Vasconcelos Goncalves wrote:
> 
> Olá, preciso de ajuda nesta questão, se possível ainda para hoje...
> desde já agradeço vossa ajuda,
>  
> Prove que:
> cos(Pi/65)*cos(2Pi/65)*cos(4Pi/65)*cos(8Pi/65)*cos(16Pi/65)*cos(32Pi/65)=1/64



Re: Gavetas

2000-12-08 Por tôpico Augusto Morgado

O fato de a equaçao em x ter uma unica raiz nao equivale a equaçao em y
ter uma unica raiz.
Veja, se a equaçao em y tiver uma raiz positiva e uma negativa, a
equaçao em x tera uma unica raiz real.
Na verdade o problema eh determinar a para que a equaçao tenha uma unica
raiz positiva. 

Rodrigo Villard Milet wrote:
> 
> Para o seu problema :
> Seja 2^x = y ( y > 0). Daí, y^2 - a*y + (3a-8) = 0 (I)
> Delta = D.. D = a^2 - 12a + 32.  Como queremos única solução, devemos
> ter a = 4 ou a = 8 ( Delta = 0 ). Para isso, teremos y = a/2, o que nos dá y
> = 2 e y = 4
> Daí, no primeiro caso, x = 1. Do segundo, x = 2. Parece que os valores de a
> são 4 e 8, no entanto estou meio desconfiado de alguma armadilha !
> 
> -
> Por que acho isso ?? Porque testando a = 8/3, temos 2^x ( 2^x - 8/3 ) = 0 e
> como 2^x nunca é zero, temos que x é igual a log 8/3 na base 2, que é real e
> é único ! Hum.. acho que tive uma idéia !! Como o gráfico da equação (I)
> tem a concavidade para cima e queremos uma raiz real apenas, basta obrigar
> y1 < 0 < y2 ( y1 e y2 são raízes) , pois y1 < 0 gera 2^x < 0... absurdo !
> Com isso, defino P(y) = y^2 - a*y + (3a-8). Para termos o 0 entre as raízes,
> devemos ter P(0) < 0, o que nos dá 3a-8 < 0 ... a < 8/3. Como verificamos
> que 8/3 serve tb, então a=<8/3. Daí, temos a solução seguinte :
>  a pertence aos reais, tal que a=4 ou a=8 ou a=<8/3 ( o que parece agora
> estar certo )
> 
> Abraços,
>¡ Villard !
> 
> -Mensagem original-
> De: Eduardo Favarão Botelho <[EMAIL PROTECTED]>
> Para: [EMAIL PROTECTED] <[EMAIL PROTECTED]>
> Data: Quinta-feira, 7 de Dezembro de 2000 23:21
> Assunto: Gavetas
> 
> >Olá a todos!
> >
> >O problema a seguir saiu da Eureka 5, do princípio  das gavetas, e sua
> >solução pode ser simples, mas empaquei nela. Dêem uma olhada, por favor:
> >
> >Mostre que para qualquer coleção de n inteiros há um subconjunto cuja
> >soma é divisível por n.
> >
> >Bom... e aproveitando, vou deixar um problema que achei muito
> >interessante de uma apostila do curso pré-vestibular do Objetivo:
> >
> >Considere, em R, a equação 4^x - a.2^x + (3a - 8) = 0
> >
> >Determine todos os valores de a para que a equação admita uma única
> >solução real.
> >
> >Abraços, Eduardo
> >
> >



Re: divisibilidade

2000-12-06 Por tôpico Augusto Morgado

Nossa! Achei que, provado que os fatores primos eram os mesmos,
completar a prova era infantil. Nao eh nao.
Suponha que os expoentes do fator primo p em a e b sao respectivamente s
e t.
Como a divide b^2, s<2t (< significa menor ou igual). Bomo b^2 divide
a^3, 2t<3s ..

No fim encontra-se
s<2t<3s<4t<5s
Vamos mostrar que t/s (que chamarei de r) eh igual a 1.
Dividindo as desigualdades por s encontra-se
1<2r<3<4r<5<6r..
1/2 
> Salvo melhor juizo,
> o fato de a e b terem os mesmos fatores primos, nao significa que sejam
> iguais.
> Creio que a=2^8 e b=2^9 constituem um contra-exemplo.
> a=2^8 | b^2=2^18 | a^3=2^24 | b^4=2^36 | a^5=2^50
> JP
> 
> -Mensagem original-
> De: Augusto Morgado <[EMAIL PROTECTED]>
> Para: [EMAIL PROTECTED] <[EMAIL PROTECTED]>
> Data: Quarta-feira, 6 de Dezembro de 2000 10:18
> Assunto: Re: divisibilidade
> 
> >Se a=1, b^2 divide a^3 implica b^2 divide 1 e como b eh positivo, b=1.
> >Se b=1, a divide b^2 implica a divide 1 e como a eh positivo, a=1.
> >Logo, basta provar nos casos a e b maiores que ou iguais a 2.
> >Se p eh um fator primo de a, p eh tambem de b^2 e portanto de b.
> >Se p eh um fator primo de b, p eh tambem de b^2 e portanto de a^3 e
> >portanto de a.
> >Logo, os fatores primos de a e b sao os mesmos.
> >
> >Marcelo Souza wrote:
> >>
> >> Oi pessoal!
> >>
> >> Alguém poderia resolver o problema abaixo para mim
> >>
> >> 1. Sendo a e b inteiros positivos tais que a|b^2, b^2|a^3, a^3|b^4,
> >> b^4|a^5.., prove que a=b.
> >>
> >> agradeço antes
> >> abraços
> >> marcelo
> >>
> >>
> 
> _
> >> Get more from the Web.  FREE MSN Explorer download :
> http://explorer.msn.com
> >



Re: divisibilidade

2000-12-06 Por tôpico Augusto Morgado

Se a=1, b^2 divide a^3 implica b^2 divide 1 e como b eh positivo, b=1.
Se b=1, a divide b^2 implica a divide 1 e como a eh positivo, a=1.
Logo, basta provar nos casos a e b maiores que ou iguais a 2.
Se p eh um fator primo de a, p eh tambem de b^2 e portanto de b.
Se p eh um fator primo de b, p eh tambem de b^2 e portanto de a^3 e
portanto de a.
Logo, os fatores primos de a e b sao os mesmos.

Marcelo Souza wrote:
> 
> Oi pessoal!
> 
> Alguém poderia resolver o problema abaixo para mim
> 
> 1. Sendo a e b inteiros positivos tais que a|b^2, b^2|a^3, a^3|b^4,
> b^4|a^5.., prove que a=b.
> 
> agradeço antes
> abraços
> marcelo
> 
> _
> Get more from the Web.  FREE MSN Explorer download : http://explorer.msn.com



Re: binomio

2000-12-04 Por tôpico Augusto Morgado

n!= n (n-1) (n-2)... (n-k+1) [(n-k)(n-k-1)...1]=
 n (n-1) (n-2)... (n-k+1)[(n-k)!]
 

Marcelo Souza wrote:
> 
> Oi pessoal!
> Olha, eu estava vendo uma lista de problemas e vi a resolução de um deles.
> Mas usava uma notação pra formula de combinação que eu nunca vi.
> n!/(n-k)!k!, esta seria a que eu sempre vejo. Mas a que estava no livro era
> assim:
> n(n-1)(n-2)...(n-k+1)/k!
> Tentei ficar demonstrando que era iguais, mas fiquei dando voltas e voltas
> na mesma coisa. Alguém poderia mostrar pra mim que essa ultima é igual
> aquela primeira.
> Obrigado
> abraços
> marcelo
> _
> Get more from the Web.  FREE MSN Explorer download : http://explorer.msn.com



Re: Parte inteira

2000-12-02 Por tôpico Augusto Morgado

Na realidade essa soluçao eh natural. A desigualdade inicial eh obtida
comparando a area entre y= 1/raiz de x entre x=n e x=n+1 com areas de
retangulos inscrito e circunscrito.



Re: Volume do cubo

2000-12-01 Por tôpico Augusto Morgado

Ta faltando coisa. Nao seria, dada a soma das arestas, o volume eh
maximo quando  ou 
dada a area, o volume eh maximo quando...?

> Davidson Estanislau wrote:
> 
>  
>     Como podemos demonstrar, sem usar o m?todo de Lagrange, que um
> volume de um paralelep?pedo ? m?ximo, quando suas arestas tem as
> mesmas dimens?es ? (ou seja quando o paralelep?pedo ? um cubo).
>  
>     Davidson



Re: Probleminhas

2000-12-01 Por tôpico Augusto Morgado



Marcos Eike wrote:
> 
> Se a resposta estiver correta, podemos solucionar seu problema deste modo:
> 
> > * Três estudantes de Matematica, de passeio por uma cidade, notaram que o
> > condutor de um automovel infringira o regulamento de transito. Nenhum dos
> > estudantes lembrava do numero (de quatro algarismos obviamente) da licença
> > do carro, mas, como os tres eram matematicos, cada um deles havia
> registrado
> > alguma particularidade desse numero. Um deles notara que os dois primeiros
> > algarismos eram iguais. O segundo percebera que tambem os dois ultimos
> eram
> > iguais. E, quanto ao ultimo, garantia ele que o numero inteiro era um
> > quadrado perfeito. Qual era o numero da placa ?
> 
> A = aacc => 10^3a + 10^2a + 10c + c => 1100a + 11c = x^2 => 11(100a + c) =
> x^2
> 
> x^2 é múltiplo de 11.
> 
> a+a - (c+c) = 11t (POR QUÊ?)
> 2a - 2c = 11t
> 2(a-c) 11t => a-c é mutiplo de 11. => a=c logo:
> 
> 11(101a) = x^2 => a = 0
> 
> Assim a placa é 



Re: questao do ITA

2000-12-01 Por tôpico Augusto Morgado

E
Vou chamar P^-1 de Q. Como QP=I, temos
det (kI-B) =det (QkP-QAP)= det[Q(kI-A)P]=
detQ.det(kI-A).detP=det(kI-A).detQ.detP=
det(kI-A).det(QP)=det(kI-A).detI=det(kI-A)

Eduardo Quintas da Silva wrote:
> 
> Dizemos que duas matrizes n x m, A e B sao semelhantes se existe uma
> matriz n x n inversível P tal que B = (P^-1).A.P. Se A e B sao matrizes
> semelhantes quaisquer, entao
> 
> a) B e sempre inversivel
> b) Se A e simetrica, entao B tambem e simetrica
> c) B^2 e semelhante a A
> d) Se C e semelhante a A, entao BC e semelhante a A^2
> e) det(kI - B) = det(kI - A), onde k = numero real qualquer
> 
> P^-1 = matriz inversa de P
> A^2 = A.A



Re: Curiosidade

2000-12-01 Por tôpico Augusto Morgado

Fui (no ginásio).
Acho que nao tem nada a ver.

Eduardo Favarão Botelho wrote:
> 
> Olá a todos!
> 
> Escrevo esta msg  por uma curiosidade. Supondo que todos da
> lista tem familiaridade com a Matemática, pergunto: são todos bons de conta
> de cabeça? Ser bom numa coisa leva à outra? Será mito ou será verdade?
> Conto com a participação de vcs, dizendo de si próprios se são bons não
> só no papel, mas tb de cuca (Nicolau e Ralph, por favor, manifestem-se:
> afinal, vcs são nossos grandes campeões!!!).
> 
> Abraços, Eduardo



Re: Probleminhas

2000-12-01 Por tôpico Augusto Morgado

So um pequeno comentario. O primeiro problema (nao com esses numeros e
sim literal) foi discutido por Newton e eh conhecido como "o problema
dos bois de Newton".

Biscoito wrote:
> 
> Foi mal, mas não faço idéia de como fazer o primeiro
> exercício. Fiquei muito curioso até, pq até se poderia
> fazer de forma braçal, mas é interessante saber a
> forma correta ALLguém?
> 
> --- Ali Ahmad Smidi <[EMAIL PROTECTED]> wrote:
> > * O capim cresce no pasto todo com igual rapidez e
> > espessura. Sabe-se que 70
> > vacas o comeriam em 24 dias, e 30 em 60. Quantas
> > vacas comeriam todo o capim
> > em 96 dias ?
> 
> Já o segundo, gostaria q me avisassem caso haja um
> modo mais rápido de se resolucioná-lo.
> 
> Tomando o número com 4 algarismos a, b, c, d, o número
> sairia assim: abcd. a=b e c=d, então vemos q o número
> fica então aacc. Agora vem a parte q acho q deve haver
> um método melhor. Eu resolvi vendo todos os quadrados
> perfeitos de 1 a  (quadrado de 1 a 99).
> Obviamente, muitos eu pulei já q, por exemplo, o
> quadrado dos números entre 10 e 31 resulta num número
> do formato 0xyz. Portanto, não foi tão braçal assim.
> Dividi os quadrados perfeitos na casa dos milhares e
> analisei quanto aos dois primeiros algarismos. Até o
> primeiro milhar (00cd) há os quadrados de 1 a 9 ("cd"
> seria 01, 04, 09, 16, 25, 36, 49, 64, 81). No segundo
> milhar (11cd) há o quadrado de 34 (1156) e só.
> Terceiro milhar, 47 (2209). Quarto, 58 (3364). Quinto,
> 67 (4489). No sexto milhar, não há, pois 74(5476) e
> 75(5625). Sétimo, tb não há, pois 81(6561) e 82(6724).
> No oitavo, encontra-se o resultado, 88 (7744).
> Encontrei o quadrado de 0 () tb e gostaria de
> saber se isso se qualifica como uma solução.
> Por favor, se houver alguma forma menos pedreira q
> essa, avisem-me pq isso numa prova não seria muito
> produtivo.
> 
> Vik
> 
> =
> "Meu Deus, protegei-me de meus amigos!
> Dos meus inimigos eu me encarregarei."
> 
>   Voltaire
> 
> __
> Do You Yahoo!?
> Yahoo! Shopping - Thousands of Stores. Millions of Products.
> http://shopping.yahoo.com/



Re: =?x-user-defined?q?An=E1lise=20combin=E1toria=2E?=

2000-11-30 Por tôpico Augusto Morgado

Em relaçao ao ultimo problema, ha 3 categorias de cartoes:
1) os que virados de ponta-cabeça (essa foi em homenagem 
aos paulistas, ja que hoje eh dia de Santo Andre)se tornam
coisas sem sentido, tipo 35608.
2) os que virados de cabeça para baixo se transformam em 
numeros diferentes, tipo 88069
3) os que virados de cabeça para baixo se transformam em 
si proprios, tipo 69069
A classe 2 eh a que permite economia.
Eh facil calcular
1+2+3
2+3
3
Trabalhem, meninos!



Carlos Stein Naves de Brito wrote:
> 
> > From: "Marcos Paulo" <[EMAIL PROTECTED]>
> > Reply-To: [EMAIL PROTECTED]
> > Date: Tue, 28 Nov 2000 23:45:14 -0200
> > To: <[EMAIL PROTECTED]>
> > Subject: Re: Análise combinátoria.
> >
> >
> > - Original Message -
> > From: mcddj <[EMAIL PROTECTED]>
> > To: <[EMAIL PROTECTED]>
> > Sent: Tuesday, November 28, 2000 9:25 PM
> > Subject: Análise combinátoria.
> >
> >
> >> Preciso de ajuda, urgente. Quem puder me socorrer
> >> agradeço.
> >>
> >>
> >> 1. Em um corredor há 900 armários, numerados de 1 a 900,
> >> inicialmente todos fechados. 900 pessoas, numeradas de 1
> >> a 900, atravessam o corredor. A pessoa de número k
> >> reverte o estado de todos os armários cujos números sâo
> >> múltiplos de k. Por exemplo, a pessoa de número 4 mexe
> >> nos armários de números 4, 8, 12,..., abrindo os que
> >> encontra fechados e fechando os que encontra abertos. ao
> >> final, quais armários ficarão abertos?
> >
> > Basta verificar que a porta ficará aberto somente se for "mexido" uma
> > quantidade ímpar de vezes.
> > O Armario será mexido tantas vezes quantos forem seus divisores. então
> > ficarão abertos os armarios
> > com quantidade ímpar de divisores.
> >
> > Número de divisores de um número dado:
> >
> > N = a ^x * b^y * c^z ...   com a, b, c... primos e x, y, z... pertencente
> > aos naturais
> > d(N) = (x+1)(y+1)(z+1)...
> >
> > Queremos então que o produtos dos consecutivos dos expoentes dos números
> > seja ímpar. Isso soh
> > ocorrerá se todos esses concecutivos forem ímpares e portanto os expoentes
> > devem ser PARES.
> >
> > portanto os armarios que ficarão abertos são os de número igual a um
> > quadrado perfeito.
> >
> >
> >
> >> 2. Um vagão de metrô tem 10 bancos individuais, sendo 5
> >> de frente e 5 de costas. De 10 passageiros, 4 preferem
> >> sentar de frente, 3 preferem sentar de costas e os
> >> demais não têm preferência. De quantos modos eles podem
> >> se sentar, respeitadas as preferências?
> >
> > C5,4 * C5, 3 * 3!
> >
> > Acho q eh isso ...
> nao é combinacao, pois se mudar de posicao, muda o exemplo, é so trocar C
> por arranjo, se nao me engano.
> >
> >
> >
> >> 3. De um baralho comum de 52 cartas, sacam-se
> >> sucessivamente e sem reposição duas cartas. De quantos
> >> modos isso pode ser feito se a primeira carta deve ser
> >> de copas e a segunda não deve ser um rei?
> > Dividindo em 2 casos:
> >
> > 1 Rei de copas e qq carta diferente de rei => 1 * 48
> > carta de copas (sem rei) e qq carta diferente de rei => 12 * 48
> >
> se tirou uma carta diferente de rei, sobram 47 cartas diferentes de rei,
> logo em vez de 12*48 seria 12*47
> > A resposta seria a soma ...
> >
> >
> >> 4. Escrevem-se números de 5 dígitos, inclusive os
> >> começados em 0, em cartões. Como 0, 1, e 8 não se
> >> alteram de cabeça para baixo e como6, de cabeça para
> >> baixo, se transforma em 9 e vice-versa, um mesmo cartão
> >> pode representar dois números (por exemplo, 06198 e
> >> 86190). Qual é o número mínimo de cartões para
> >> representar todos os números de 5 dígitos?
> >
> >
> >
> >
> >>
> >> Abraços...
> >>
> >>
> >
> >
> > []'s MP
> >>
> >> __
> >> Preocupado com vírus? Crie seu e-mail grátis do BOL com antivírus !
> >> http://www.bol.com.br
> >>
> >>
> >>
> >



Re: =?x-user-defined?q?Equa=E7=E3o?=

2000-11-29 Por tôpico Augusto Morgado

Gente, o algebrismo mais simples me parece que eh
1998y+1998x=xy
xy-1998x-1998y=0
xy-1998x-1998y+1998^2=1998^2
(x-1998)(y-1998)=1998^2
x-1998 deve ser divisor de 1998^2
etc
Ralph Costa Teixeira wrote:
> 
> Eduardo Favarão Botelho wrote:
> >
> > Olá Ralph!
> >
> > é verdade... foi um deslize.  Este problema, no entanto, me suscitou
> > algumas dúvidas a respeito da existência da equação. No caso de  1/x + 1/y =
> > 1/1998,  x pode ser zero? Porque se for, 1/x não existe. Como fica, então, o
> > y? Vale 1998? Veja que eu tentei excetuar o caso em que que y e x davam zero
> > justamente por achar que não dava.
> 
> Oi, Eduardo.
> 
> Você está corretíssimo em dizer que x=0 ou y=0 não valem. A sua solução
> original, em particular excluía corretamente o caso
> x - 1998 = -1998 exatamente por isso.
> 
> No entanto, esta solução nova já AUTOMATICAMENTE elimina tais casos
> pois procuramos apenas os divisores POSITIVOS de 1998^2. Tais divisores
> são os valores de x-1998 Assim, o caso x=1998 (x-1998=0) não
> aparecerá, nem x=0 (x-1998= -1998) (nem 0 nem -1998 estão na nossa lista
> dwe divisores).
> 
> Por outro lado, x-1998=1998 não é algo que se deva excluir. Este caso
> gera x=2.1998 e y=2.1998, que é uma solução válida do problema.
> >
> > > Há 3.7.3 = 63 deles, e todos são válidos para este
> > >problema.
> >
> > Do mesmo modo, e se x = 1998? (suponho que aconteça para y =0)
> 
> Abraço,
> Ralph



Re: Um limite muito dificil

2000-11-29 Por tôpico Augusto Morgado

ILEGÍVEL

> Jorge Peixoto Morais wrote:
> 
> A_(i-1)=k, e A_i= [(n-1)k +C/(k^(n-1)]/n.
> Prove que essa sequencia tende a (C)^(1/n) quando i tende a infinito
> (a_0 eh qualquer real diferente de zero).
> Eu usei "k" soh para nao escrever A_(I-1) na formula, o que a deixaria
> confusa.



Re: EXERC:

2000-11-28 Por tôpico Augusto Morgado

f(1) pode ser escolhido de n-1 modos.
f(2) pode ser escolhido de n-2 modos.
.
A resposta eh
(n-1)(n-2)...(n-m)

Carlos Stein Naves de Brito wrote:
> 
> Sejam os conjuntos Dm(1,2,...,m) e In(1,2,...,n), com n>m. Temos a funcao f:
> Dm -> In,  tal que todo elemento mi de Dm leve a um elemento nj de In, sendo
> nj>mi (> é maior que). Diga quantas funcoes f admitem a condicao.



Re: Velocidade (em funcao do tempo) de um corpo sob acao de uma forca variavel

2000-11-27 Por tôpico Augusto Morgado

Realmente, isso dah uma equaçao diferencial de segunda ordem

> Jorge Peixoto Morais wrote:
> 
> Eu estava tentando descrever o movimento de um corpo atrelado a uma
> mola ideal deformada;
> usando Calculo Integral, eh facil calcular a velocidade do corpo em
> funcao da posicao, mas calcular velocidade, aceleracao e posicao em
> funcao do tempo parece ser bem mais dificil!Ou eu nao sou
> suficientemente esperto ou preciso aprender mais Calculo
> (provavelmente a primeira alternativa...)...
> Podem me ajudar?



Re: Velocidade (em funcao do tempo) de um corpo sob acao de uma forca variavel

2000-11-27 Por tôpico Augusto Morgado

PS: Eh claro que eh a segunda alternativa,4,.

> Jorge Peixoto Morais wrote:
> 
> Eu estava tentando descrever o movimento de um corpo atrelado a uma
> mola ideal deformada;
> usando Calculo Integral, eh facil calcular a velocidade do corpo em
> funcao da posicao, mas calcular velocidade, aceleracao e posicao em
> funcao do tempo parece ser bem mais dificil!Ou eu nao sou
> suficientemente esperto ou preciso aprender mais Calculo
> (provavelmente a primeira alternativa...)...
> Podem me ajudar?



Re: Demostre!!!!!!!!!!!!!!!!!

2000-11-26 Por tôpico Augusto Morgado

Ah, agora sim. Creio que dos que participam ativamente da lista, o Ralph
é o especialista.

Marcos Eike wrote:
> 
> Eu gostaria de saber como demonstrar as curvas de Bézier, concomitantemente,
> a tecnologia NURBS, que atualmente são explorados pelos melhores softwares
> de computação gráfica do mercado.
> 
> Ats,
> Marcos Eike



Re: Demostre!!!!!!!!!!!!!!!!!

2000-11-26 Por tôpico Augusto Morgado

Eis o que eu chamo de um e-mail claro!
Entendi tudo.
Morgado

> Marcos Eike wrote:
> 
> Pessoal, demostre para mim da forma mais clara possível a função de
> Bézier e se não for incômodo poderiam me falar mais sobre o NURBS?
> 
> Ats,
> Marcos Eike
>  



Re: Radioatividade

2000-11-25 Por tôpico Augusto Morgado

Embora a explicaçao de JP esteja otima (como tudo o que ele faz), vou
aproveitar a chance oferecida pelo Jorge na parte 2 do seu correio.
Conforme Jorge disse 1/2 dos atomos vivem entre 0 e T (aqui T eh a meia
vida), 1/4 vivem entre T e 2T, 1/8 vivem entre 2T e 3T, etc.
A vida media e algo entre 0.1/2+T.1/4+ 2T.1/8+... = T e
T.1/2+2T.1/4+3T.1/8+...=2T.
Para calcular cada uma dessas somas basta chama-la de S, calcular S/2 e
subtrair. Obtem-se uma PG.

Portanto, a vida media eh aproximadamente 1,5T=
1,5.ln2/k que eh aproximadamente 1,04/k. Eh claro que usamos uma
aproximaçao grosseira , mas deve servir para o Jorge se convencer que
nossa intuiçao de que a vida media e mais curta (eh a mesma intuiçao que
eu tinha quando estudava Quimica no cientifico) esta errada. 
Morgado
> José Paulo Carneiro wrote:
> 
> Pode-se interpretar esta "vida media" como uma media ponderada: uma
> especie de tempo "vivido", ponderado pela proporcao de massa existente
> m(t)/m(0) durante esse tempo.
> Se voce dividir o intervalo de tempo [0;T] em n subintervalos de mesmo
> comprimento deltat=T/n, imaginar que nesses intervalos m nao varia
> muito e portanto pode ser aproximado por seu valor em um ponto
> qualquer do intervalo, a media ponderada de que falei serah a soma dos
> produtos de m(t)/m(0) por deltat de 0 a T. Fazendo deltat tender a 0,
> isto eh a integral de 0 a T de (m(t)/m(0)) dt, que eh: [1 -
> exp(-kT)]/k.
> Fazendo agora T tender a infinito, isto dah 1/k.
> JP
>  
> -Mensagem original-
> De: Jorge Peixoto Morais <[EMAIL PROTECTED]>
> Para: [EMAIL PROTECTED] <[EMAIL PROTECTED]>
> Data: Sexta-feira, 24 de Novembro de 2000 17:35
> Assunto: Radioatividade
> 
> Duas perguntas:
> 1: Como se chega a "vida media=1/k"?
> 2: Ora, como a meia vida porde ser 70% da vida media? Isso eh confuso;
> a mim parece que o mais provavel seria que a meia-vida fosse bem mais
> curta, pois haveria uma chance grande de a substancia decair metade
> dos atomos varias vezes, ou seja, a vida media deveria ser de (varias
> vezes)x meia-vida. O que eu quis dizer eh que o mais provavel seria
> que o atomo ficasse com massa m/2 (depois de uma meia-vida), depois
> m/4, m/8, depois m/16 e soh chegasse a 0m depois de muitas
> meia-vidas...



Re: =?x-user-defined?q?Combinat=F3ria?=

2000-11-25 Por tôpico Augusto Morgado

Outro modo de fazer seria: Dos 7 lugares, devemos escolher tres para
colocar os quatros, o que pode ser feito de C7,3=35 modos. Dos quatro
lugares que sobraram, devemos escolher dois para botar os oitos, o que
pode ser feito de C4,2=6 modos. Agora temos duas casas a preencher, o
que pode ser feito de 8x8=64 modos e a resposta seria 35x6x64=13440
modos. Devemos descontar os começados em 0,que sao
C6,3 x C3,2 x 8 = 20x3x8=480 e a resposta eh 12960.
Como as respostas deram diferentes, uma estah certa e a autra nao. Qual
delas e onde estah o erro?

"Alexandre F. Terezan" wrote:
> 
> Imagine o número 44488XY de 7 dígitos, onde X é um algarismo diferente de 4
> e 8.
> 
> 1o caso: X diferente de Y
> 
> Nessa situacao, há 7!/(3! x 2!) = 420 modos de dispormos os algarismos
> (anagramas de "44488XY").
> 
> Além disso, há 8 possibilidades para X (X diferente de 4 e 8) e 7
> possibilidades para Y (Y diferente de X, 4 e 8).
> 
> Logo, temos 420 x 7 x 8 = 23520 possibilidades.
> 
> Deve-se desconsiderar os casos em q o primeiro algarismo é zero. Existem
> 6!/(3! x 2!) modos de arrumarmos "44488A", onde A diferente de 0, 4 e 8: 60
> x 7 = 420 maneiras onde 0 é o primeiro algarismo.
> 
> Assim, há 23100 maneiras de dispormos 44488XY.
> 
> 2o caso: X = Y
> 
> Aqui, temos 7!/(3! x 2! x 2!) = 210 maneiras de dispormos "44488XX". Como X
> diferente de 4 e 8, há 8 "X" possíveis, nos dando 1680 casos.
> 
> Desses 1680, tiremos os casos onde o primeiro algarismo é zero. Neste caso
> há 6!/(3! x 2!) possibilidades de arrumarmos "444880" a partir do primeiro
> zero, o q nos dá 60 casos impossíveis.
> 
> Logo, 1620 casos satisfazem, quando X = Y.
> 
> TOTAL: 23100 + 1620 = 24720 possibilidades.
> 
> - Original Message -
> From: "ricardopanama" <[EMAIL PROTECTED]>
> To: <[EMAIL PROTECTED]>
> Sent: Sexta-feira, 24 de Novembro de 2000 17:58
> Subject: Combinatória
> 
> Agrdeço a quem responder este problema de combinatória:
> 
> Quantos são os algarismos de 7 dígitos nos quais o
> algarismo 4 figura exatamente 3 vezes e o algarismo 8
> exatamente 2 vezes?
> 
> Abrços.
> 
> __
> Preocupado com vírus? Crie seu e-mail grátis do BOL com antivírus !
> http://www.bol.com.br



Re:

2000-11-24 Por tôpico Augusto Morgado

Veja, basta escolher no contradominio os m elementos que farao parte da
imagem. Escolhidos esses elementos, o 1 serah ligado ao menor dos
escolhidos, o 2 serah ligado ao segundo menor etc pois a funçao deve ser
estritamente crescente. O numero de modos de 
escolher m elementos dentre n eh Cn,m = n!/m!(n-m)!


O icosaedro tem 20 faces triangulares, 12 vertices e 30 arestas. Uma
diagonal liga dois vertices (o numero de modos de escolher dois vertices
para ligar eh C12,2 = 66) nao pertencentes a mesma face. Ora, na nossa
contagem de 66 incluimos as ligaçoes de vertices de uma mesma face (que
sao as arestas; ou diagonais das faces, se existissem, o que não eh o
caso pois as faces sao triangulares). A resposta eh 66-30=36.

Uma diagonal de um prisma liga obrigatoriamente um vertice da face "de
cima" a um da "de baixo". Ha 6 modos de escolher o vertice da face de
cima e, depois disso, 3 de escolher o da de baixo sem formar aresta ou
diagonal de face.A resposta eh 18.



tatiania wrote:
> 
> Preciso de ajuda, quem me ajudar agradeço.
> 
> 1) Sejam Im = {1,2,...,m} e In = {1,2,...,n}, com m
> menor ou igual a n. Quantas são as funções f: Im --> In
> estritamente crescente?
> 
> 2) Quantas diagonais possui:
> 
> a) um icosaedro regular?
> 
> b) um prisma hexagonal?
> 
> Abraços.
> 
> 
> __
> Preocupado com vírus? Crie seu e-mail grátis do BOL com antivírus !
> http://www.bol.com.br



Re: Radioatividade

2000-11-20 Por tôpico Augusto Morgado

Como escreveu o José Paulo, a meia-vida é ln2/k.
ln2 vale aproximadamente 0,693 e eh arredondada para 0,7 em todos os
livros de Quimica. Portanto a meia-vida eh aproximadamente 9,7 isto eh
70$ de 1/k. 1/k eh o que se chama de vida media



Re: probabilidade

2000-11-20 Por tôpico Augusto Morgado

Terrivel.
Provavelmente o autor da questao desejava que a prob. de cara fosse 65%
e a de coroa 35%, o que daria para resposta 0,65x0,65=0,4225=42,25%.
Mas isso está errado. 65% não eh 30% maior que 35%.
As probs sao na verdade a e 1,3a. Como a soma eh 1, a=1/2,3 e as probs
sao 1/2,3 e 1,3/2,3.
A resposta eh 1,3/2,3 ao quadrado, o que dah aproximadamente 31,95%.

> filho wrote:
> 
> Uma moeda ? viciada de modo que a probabilidade de ocorrer cara numa
> jogada ? 30% a mais do que a de ocorrer coroa. Se essa moeda for
> jogada duas vezes consecutivamente, determine a probabilidade de
> ocorr?ncia de cara nas duas jogadas.



Re: =?x-user-defined?q?Quest=E3o?= do ITA - Ajuda

2000-11-17 Por tôpico Augusto Morgado



Thomas de Rossi wrote:
> 
> Oi Pessoal,
> Segue abaixo uma questão do ITA (muito antiga), de Física. Acredito que a
> Física não seja tão comprometedora (já que está não é uma lista de Física),
> sobrando assim muita matemática...
> 
> Eu tentei resolver mas não chego a resposta final dada como certa, vejam a
> questão;
> 
> ITA-FIS) Um móvel 'A' parte da origem 'O', com velocidade inicial nula, no
> instante 't0 = 0 s', e percorre o eixo 'Ox' com aceleração constante 'a'.
> Após um intervalo de tempo 'deltat', contado a partir da saída de 'A', um
> segundo móvel 'B' parte de 'O' com uma aceleração igual a 'na', sendo 'n>1'.
> 'B' alcançara 'A' no instante:
> 
> A resposta final deverá ser: t = (raiz(n)/ (raiz(n) - 1))* deltat.
> 
> Dêem uma olhada nos meus cálculos...
> 
> Primeiramente referenciaremos o movimento de acordo com as acelerações e o
> tempo em que os móveis 'A' e 'B' saem do repouso.
> As funções são:
> 'aA(t) = a', se t > 0 e 'aB(t) = na', se t > deltat
> Assim para função das velocidades,
> As funções são:
> vA(t) = a*t, e vB(t) = an*t - an*deltat
> Sendo as funções posição definidas como:
> vA(t)*t = a*t^2, e vB(t)*t = an*t^2 - an*deltat*t
> Ficando assim:
> A(t) = at^2
> B(t) = na*t^2  - na*deltat*t
> 
> No encontro dos móveis teremos: A(t) = B(t),  ou vA(t) = vB(t), assim o
> tempo 't' do encontro será igual a,
> 
> vA(t) = a*t, e vB(t) = an*t - an*deltat,
> 
> vA(t) = vB(t),
> a*t = an*t - an*delta*t,
> t = n*t - n*deltat,
> t - nt = - n*deltat,
> (1 - n)*t = - n*deltat,
> t = - n*deltat / (1 - n)
> 
> Não estou conseguindo fechar nos cálculos e não sei se é por alguma dedução
> Física/ou Matemática que não estou conseguindo chegar a resposta correta.
> 
> Agradeço pela ajuda,
> 
> Abraços, Thomas.
> 
> _
> Get Your Private, Free E-mail from MSN Hotmail at http://www.hotmail.com.
> 
> Share information about yourself, create your own public profile at
> http://profiles.msn.com.



Re: Eureka

2000-11-15 Por tôpico Augusto Morgado

1564-4.
Eh a mesma agencia, o Banco do Brasil adora trocar os numeros de suas
agencias.
Morgado

Carlos Stein Naves de Brito wrote:
> 
> Qual a verdadeira Agencia para depositar:
> 1564-4 ou 0598-3?? uma ta na internet outra na eureka 6



Re: Parte Inteira

2000-11-15 Por tôpico Augusto Morgado



Augusto Morgado wrote:
> 
> Experimente abrir em casos, conforme x/2 esteja entre um inteiro e ele
> mais 0,5 ou entre um inteiro e um inteiro mais(ERA MENOS< DROGA) 0,5.
> 
> > Rodrigo Villard Milet wrote:
> >
> > Uma vez eu vi algo muito intuitivo sobre a parte inteira de um n?mero
> > [x]. ? o seguinte :
> >  [x/2] + [(x+1)/2] = [x]
> > Como se prova isso ?? Algu?m me ajuda ?? N?o sei se ? f?cil ou n?o,
> > mas eu n?o estou conseguindo provar.
> >  Abra?os,
> >  ? Villard !



Re: Parte Inteira

2000-11-15 Por tôpico Augusto Morgado

Experimente abrir em casos, conforme x/2 esteja entre um inteiro e ele
mais 0,5 ou entre um inteiro e um inteiro mais 0,5.

> Rodrigo Villard Milet wrote:
> 
> Uma vez eu vi algo muito intuitivo sobre a parte inteira de um n?mero
> [x]. ? o seguinte :
>  [x/2] + [(x+1)/2] = [x]
> Como se prova isso ?? Algu?m me ajuda ?? N?o sei se ? f?cil ou n?o,
> mas eu n?o estou conseguindo provar.
>  Abra?os,
>  ? Villard !



Re: Duas =?x-user-defined?q?quest=F5ezinhas=21=21=21?=

2000-11-15 Por tôpico Augusto Morgado

Bonitas questoes. A primeira (com outros numeros) caiu num vestibular da
UERJ e nao houve 50 candidatos que conseguissem resolve-la.

Via Lux wrote:
> 
> Olá pessoal,
> 
> Aí vão duas questões que deixam muitos alunos confusos...
> 
> E vcs que dizem?
> 
> 1) Numa pesquisa sobre o consumo dos produtos A, B e C, obteve-se o seguinte
> resultado: 68% dos entrevistados consomem A, 56% consomem B, 66% consomem C
> e 15% não consomem nenhum dos produtos. Qual a porcentagem mínima de
> entrevistados que consomem A,B e C?
> 
> 2) Considerando que em uma festa há 15 pessoas, não podemos afirmar que:
> 
> a)pelo menos duas nasceram no mesmo mês do ano.
> 
> b)pelo menos três nasceram no mesmo dia da semana
> 
> c)se uma das pessoas conhece as demais então existem pelo menos duas com o
> mesmo número de conhecidos (o conhecer alguém é recíproco)
> 
> d)se uma pessoa não conhece ninguém então pode não existirem duas pessoas
> com o mesmo número de conhecidos (o conhecer alguém é recíproco)
> 
> e) a diferença de idade "em anos" de duas delasé um múltiplo de 14
> 
> Lembranças a todos,
> 
> Fui!
> 
> Luciano M. Filho



Re: =?x-user-defined?q?L=F3gica=3F=21?=

2000-11-13 Por tôpico Augusto Morgado

Ha um excelente texto de autoria do professor Joao Bosco Pitombeira de
Carvalho que foi publicado pela RPM.
Provavelmente Josimat, o mais organizado dos membros desta lista, ha de
indicar o numero. 
O mesmo texto faz parte do livro Analise Combinatoria e Probabilidade,
da Coleçao do Professor de Matematica, SBM.

Rodrigo Villard Milet wrote:
> 
> As alternativas a,b,d estão colocadas fora de um contexto... portanto, nada
> pode se afirmar. Na letra e, vemos sua falsidade, pois por exemplo, todas as
> pessoas poderiam fazer aniversário em dezembro. No entanto, na letra c, como
> temos mais de 12 pessoas, vemos claramente ( pelo principio de dirichlet ) q
> ela é verdadeira !
>  Abraços,
> ¡ Villard !
> -Mensagem original-
> De: Thomas de Rossi <[EMAIL PROTECTED]>
> Para: [EMAIL PROTECTED] <[EMAIL PROTECTED]>
> Data: Segunda-feira, 13 de Novembro de 2000 00:43
> Assunto: Lógica?!
> 
> >Pessoal,
> >
> >olhem só a questão abaixo...
> >
> >3) Em uma reunião, encontram-se 15 pessoas. Podemos afirmar que,
> >necessariamente,
> >
> >(a) pelo menos uma delas tem mais de 30 anos.
> >(b) pelo menos duas delas são do sexo feminino.
> >(c) pelo menos duas delas fazem aniversário no mesmo mês.
> >(d) pelo menos uma delas é brasileira.
> >(e) pelo menos uma delas nasceu em Janeiro ou Fevereiro.
> >
> >Comentário: A questão abaixo foi colocada numa prova  de vestibular, eu não
> >sei se a escolha da alternativa é mais uma questão de lógica (sentimento de
> >chute), ou se tem como determinar matematicamente qual a resposta correta?
> >
> >Qualquer ajuda será bem vinda,
> >
> >M.Obrigado,
> >Thomas.
> >
> >_
> >Get Your Private, Free E-mail from MSN Hotmail at http://www.hotmail.com.
> >
> >Share information about yourself, create your own public profile at
> >http://profiles.msn.com.
> >
> >



Re: Livros do IMPA

2000-11-10 Por tôpico Augusto Morgado

Não, mas eles podem ser comprados pelo correio. Escreva para SBM,
Estrada D. Castorina 110, Rio de Janeiro.
Se você disser onde mora, talvez se possa sugerir outra alternativa.

> Jorge Peixoto Morais wrote:
> 
> Hei, seus autores: não se pode comprar seus livros pela WEB?Eu não vi
> nada disso no site do IMPA...Eu finalmente decidi comprar um monte de
> livros, mas a minha casa fica a 50 minutos do ponto de venda mais
> proximo...



IME

2000-11-08 Por tôpico Augusto Morgado

Como muitos da lista se interessam pelas provas do IME, recomendo o site
www.gpi.g12.br
Alias, a rede esta cheia de soluçoes da prova com alguns errinhos.
Morgado



Re: Lemi

2000-11-08 Por tôpico Augusto Morgado

A lemniscata de Bernoulli eh um tipo de oval de Cassini. 
Uma oval de Cassini eh definida como o lugar dos pontos do plano cujo
produto das distancias a dois pontos fixos do plano eh constante.
Se os pontos forem (a,0) e (-a,0) e a constante for k a equaçao da oval
sera
[(x-a)^2+y^2].[(x+a)^2+y^2]=k^2.
No caso especial k=a^2 a oval eh a lemniscata, cuja equaçao eh  
(x^2+y^2)^2=2a^2(x^2-y^2).

a equaçao polar eh melhor, r^2=2a^2cos(2teta).

Essas curvas aparecem MARAVILHOSAMENTE no site da Wolfram.

Morgado





Biscoito wrote:
> 
> Alguém saberia me dizer o que vem a ser uma
> leminiscata (eu sei o formato, mas o resto...), para
> quê serve e a partir de que forma e/ou equação ela é
> formada para ter aquele formato em 8 deitado (ou sinal
> de infinito).
> 
> Vik
> 
> =
> 
> Experiência não é o que acontece com um homem,
> 
> mas o que um homem faz com o que lhe acontece.
> 
>   Aldous Huxley
> 
> __
> Do You Yahoo!?
> Thousands of Stores.  Millions of Products.  All in one Place.
> http://shopping.yahoo.com/



Re: Mensagem truncada do Luiz Lopes

2000-11-08 Por tôpico Augusto Morgado

Apoio pleno e irrestrito ao pedido do Jorge.
Morgado

> Jorge Peixoto Morais wrote:
> 
> Hei, Luiz! Sua mensagem veio com uns simbolos muito estranhos... tente
> escrever no modo texto, sem acentos ou simbolos matematicos, para que
> nada ocorra de errado!Sua mensagem parece ter sido bme interessante,
> mas nao posso entende-la...



Re: =?x-user-defined?q?Equa=E7=E3o?=

2000-11-06 Por tôpico Augusto Morgado



Carlos Stein Naves de Brito wrote:
> 
> Antes de extrair a raiz x-1ésima separei os casos x=1(que leva a uma
> solucao trivial sem ter que se extrair a raiz x-1esima) e x diferente
> de 1, nesse pode-se extrair a raiz, chegando a esse ponto da
> equacao...
> 
>  From: [EMAIL PROTECTED] (Hugo Iver Vasconcelos Goncalves)
>  Reply-To: [EMAIL PROTECTED]
>  Date: Tue, 31 Oct 2000 22:40:42 -0200
>  To: <[EMAIL PROTECTED]>
>  Subject: Re: Equação
> 
>  Não sei se o que eu notei está coreto mas veja:
> 
>  2=x^1/x-1 , como o denominador tem que ser diferente de 0
>  entao x diferente de 1, que é uma das respostas da equação.
>  Como isso pode acontecer?
> 
>   -Mensagem Original-
>   De: Carlos Stein Naves de Brito
>     
>   Para: [EMAIL PROTECTED]
>   Enviada em: Segunda-feira, 30 de Outubro de 2000
>   00:37
>   Assunto: Re: Equação
> 
>   Não tenho a minima ideia se está certa minha
>   solucao mas vale a tentativa:
>   Sabemos que 1 é solucao obvia, logo vamos calcular
>   com x diferente de 1 e positivo pois se nao 2x
>   seria negativo e 2^x positivo:
>   2x=2^x   
>   x=2^x / 2
>   x=2^(x-1)
>   Tirando a raiz x-1 ésima(x - 1 diferente de zero
>   pois x diferente de 1) fica:
>   x^1/x-1=2
>   (logo temos uma raiz que deve ser inteira, logo a
>   base deve ser tambem inteira), 
AQUI HA UM PROBLEMA. VOCE NAO SABE SE O X EH INTEIRO. ELE PODE SER
IRRACIONAL. SE FOSSE 3^(X-2)=X^2, QUE TEM UMA RAIZ POSITIVA IRRACIONAL,
COMO FICARIA ESSE ARGUMENTO? 

entao queremos um x
>   tal que x-1 é a potencia dos fatores de x. se
>   x=n^k temos:
>   n^k-1=k logo n^k=k+1 e temos n>=2   n^k>=2^k
>   logo k+1>=2^k, obviamente a unica solucao natural
>   é 0 e 1.
>   logo a potencia deve ser 0 ou 1. logo x-1=1, x=2,
>   ou x-1=0, x=1, que ja tinhamos.
>   logo x=1 ou 2
> 
>From: [EMAIL PROTECTED] (Hugo Iver
>Vasconcelos Goncalves)
>Reply-To: [EMAIL PROTECTED]
>Date: Sun, 29 Oct 2000 22:23:54 -0200
>To: <[EMAIL PROTECTED]>
>Subject: Equação
> 
>Alguém pode demonstrar a solução da
>equação 2^x=2x ??? É claro que a solução
>todo mundo já sabe, 1 e 2. ...



Re: =?x-user-defined?q?Polin=F4mios?=

2000-11-06 Por tôpico Augusto Morgado

As raizes de P(x)-1 (todas!) sao dadas. Entao
P(x)-1=A(x-1)(x-2)(x-3)(x-4)(x-5).
Como P(6)=0, A= -1/120.
P(0)=2.

"João Paulo Paterniani da Silva" wrote:
> 
> Olá! Alguém poderia me ahudar nesse problema de Polinômio?
> 
>Sabendo que P(x) é de quinto grau.
>I- P(1)=P(2)=P(3)=P(4)=P(5)=1
>II- P(6)=0
>III- Calcule P(0)
> 
> João Paulo Paterniani da Silva
> 
> _
> Get Your Private, Free E-mail from MSN Hotmail at http://www.hotmail.com.
> 
> Share information about yourself, create your own public profile at
> http://profiles.msn.com.



Re: =?x-user-defined?q?fun=E7=E3o?=

2000-11-05 Por tôpico Augusto Morgado

Pondo x=y=0, obtemos f(0)^2-f(0)=0. Logo, f(0)=0 ou f(0)=1.
Se f(0) = 0, fazendo y=0 obtemos x=0 para todo x, o que eh absurdo.
Logo, f(0)=1.
Fazendo y=0, temos f(x)-1=x. Logo, f(x)=x+1.
Verificando...O.K.
Marcelo Souza wrote:
> 
> oi! Alguém poderia mandar a solução do problema abaixo?
> - Determine f, se para todos os reais x e y, f(x)f(y) - f(xy) = x + y.
> Valeu
> abraços
> marcelo
> _
> Get Your Private, Free E-mail from MSN Hotmail at http://www.hotmail.com.
> 
> Share information about yourself, create your own public profile at
> http://profiles.msn.com.



Re: triangulo!

2000-11-04 Por tôpico Augusto Morgado

Bonito!
Mas uma lei dos senos no triangulo MBC daria x/senalfa=h/seno C.
Aqui alfa eh o angulo procurado e x= metade do lado AC e h=
altura=mediana. O triangulo retangulo CAP (P eh o pe da altura)
da seno C=h/2x. Isso da seno alfa=1/2.
Morgado
Douglas Coimbra de Andrade wrote:
> 
> Eis uma solucao:
> 
> 1- Trace uma paralela a BM por A, e seja S o ponto de intersecao dessa
> paralela com a reta BC (prolongamento, obviamente).
> 2-Pelos dados do problema, sejam CM=MA=x, AS=BM=z.
> 3-Pelo teorema de Tales, temos BC=BS.
> 4-Os triangulos CBM e CSA sao semelhantes pois MB//AS. Entao CM/CA=BM/AS <=>
> x/2x=z/AS, AS=2z, e alem disso o angulo CSA = angulo CBM = alfa
> 5-Observe o triangulo retangulo APS. Aplicando relacoes metricas, o seno do
> angulo ASC, que eh o angulo alfa procurado, vale sen(alfa)=AP/AS=z/2z=1/2,
> de onde conclui-se que o angulo procurado vale 30 graus.
> 
> Estou com um sono danado. Verifique meus calculos e indicacoes. Espero ter
> ajudado. Valeu!
> 
> -Mensagem original-
> De: Marcelo Souza <[EMAIL PROTECTED]>
> Para: [EMAIL PROTECTED] <[EMAIL PROTECTED]>
> Data: Sábado, 4 de Novembro de 2000 19:56
> Assunto: triangulo!
> 
> >Olá pessoal,
> >Alguém poderia mandar a solução do problema abaixo.
> >- Num triangulo ABC, a altura AP e a mediana BM são iguais. Calcule o
> angulo
> >MBC.
> >Valeu
> >Abraços
> >Marcelo
> >_
> >Get Your Private, Free E-mail from MSN Hotmail at http://www.hotmail.com.
> >
> >Share information about yourself, create your own public profile at
> >http://profiles.msn.com.
> >



Re: Geometria das Areias...

2000-11-04 Por tôpico Augusto Morgado

Wilson Areias foi um professor de geometria de muitos cursos
vestibulares do Rio de Janeiro. Faleceu prematuramente, vítima de um
acidente de carro. Foi um grande resolvedor de problemas, tendo tido
muitas de suas soluções publicadas na Crux Mathematicorum.
Acho que ele trabalhou no GPI ou no MV1 e o livro a que você se refere
deve ser a apostila do curso.
Um possível leitor dessa mensagem e que foi um grande amigo do Areias é
o Antonio Luis Santos, que deve conhecer o livro a que você se refere. 
Morgado
wrote:
> 
> Alguém ae conhece um livro de geometria q se chama
> geometria das areias ou só geometria mas q o autor
> tenha areias no nome?  e sabem se existe um livro de
> geometria do PAPELIER? se alguém souber de algo por
> favor me contate... obrigado... [EMAIL PROTECTED]
> 
> __
> Do You Yahoo!?
> Thousands of Stores.  Millions of Products.  All in one Place.
> http://shopping.yahoo.com/



Re: Desentendimentos e =?x-user-defined?q?=E2ngulo=20s=F3lido?=

2000-11-03 Por tôpico Augusto Morgado

Quanto às congruencias, vamos começar dizendo que, por se tratar de
questao de notaçao, algumas pessoas sao muito liberais a respeito e
outras nao.
Na aritmetica comum, o que eh o simetrico de x? Eh o numero que somado a
x dah zero. 
O que eh o inverso de x? Eh o numero que multiplicado por x dah 1. 
Na aritmetica modulo 7 o que eh o inverso de 4? Eh o numero que somado a
4 (modulo 7) dah zero, ou seja, eh 3. Logo voce pode escrever numa boa
que -4 = 3. 
O que eh o inverso de 2? Eh o numero que multiplicado por 2 dah 1, isto
eh, 4. Entao, 
modulo 7, 2 elevado a -1 eh igual a 4. Nao eh comum escrever isso assim:
1/2 =4 (mas os liberais nao teriam nada contra).
O 3/2 deve ser interpretado assim:
3x(inverso de 2)=-2 
Isso equivale a (multiplicando por 2)

3x1=-2x2

3x1=-4

(Vale a volta, basta multiplicar pelo inverso de 2.)

> Jorge Peixoto Morais wrote:
> 
> *Nicolau (O Grande) disse que R³ não é associativo em relação ao
> produto vetorial, mas o Ralph parece discordar! Que negócio é esse?
> *Em uma apostila estava escrito que a congruência só funciona com
> números inteiros, mas em outra eu encontrei 3/2 = -2 (mod 7), porque
> se ambos os lados forem multiplicados por 2, fica 3 = -4 (mod 7). Que
> negócio é esse?
>  
> Agora o ângulo sólido. Ângulo sólido é definido como o quociente entre
> a área (determinada pelos planos que definem o angulo e a superficie
> da esfera) de uma esfera com centro em seu vértice e a área de
> superficie total da esfera?Eu pergunto isso porque parece que o angulo
> linear foi definido assim, mas substituindo "esfera" por
> "circunferencia".



Re: =?x-user-defined?q?combinat=F3ria?=

2000-10-27 Por tôpico Augusto Morgado



Antonio Neto wrote:
> 
>Nao eram inteiros? a=-5, b=5 e c=10 eh uma solucao, acho eu. Abracos,
> olavo.
> 
> >From: "Alexandre F. Terezan" <[EMAIL PROTECTED]>
> >Reply-To: [EMAIL PROTECTED]
> >To: <[EMAIL PROTECTED]>


> >Subject: Re: combinatória
> >Date: Fri, 27 Oct 2000 00:03:10 -0200
> >
> >0 < a <= 2, pois para c > b > a > 2,a + b + c > 11 (impossível)
> >1 < b <= 4, pois para c > b > 4 >= a > 0,a + b + c > 11 (impossível)
> >Obs: b > a > 0, entao b > 1
> >
> >Assim, para:
> >
> >  I) a = 1:
> >
> >i) b = 2 --> c = 7
> >ii) b = 3 --> c = 6
> >iii) b = 4 --> c = 5
> >
> >II) a = 2:
> >
> >i) b = 3 --> c = 5
> >ii) b = 4 --> c = 4 (impossível, pois c > b)
> >
> >Logo, há 4 ternos possíveis, (1,2,7) (1,3,6) (1,4,5) e (2,3,5)
> >   - Original Message -
> >   From: Filho
> >   To: discussão de problemas
> >   Sent: Quinta-feira, 26 de Outubro de 2000 22:03
> >   Subject: combinatória
> >
> >
> >   Qual é o número de ternos (a,b,c) de números inteiros tais que a + b + c
> >= 10 e 0 < a < b < c ?
> 
> _
> Get Your Private, Free E-mail from MSN Hotmail at http://www.hotmail.com.
> 
> Share information about yourself, create your own public profile at
> http://profiles.msn.com.


Eh, mas 0 < a < b < c.



Re: senos e cossenos

2000-10-21 Por tôpico Augusto Morgado



José Paulo Carneiro wrote:
> 
> Nao eh possivel para todos os senos e cosenos.
> Para 36gr, observe que num triangulo isosceles com 36gr no vertice
> diferente, os outros angulos sao 72gr. Tracando uma bissetriz de um desses
> angulos de 72, voce obtem outro trianglinho semelhante ao grandao. Com esta
> observacao, voce mata o problema.
> JP
> 
> -Mensagem original-
> De: Marcelo Souza <[EMAIL PROTECTED]>
> Para: [EMAIL PROTECTED] <[EMAIL PROTECTED]>
> Data: Quinta-feira, 19 de Outubro de 2000 14:54
> Assunto: senos e cossenos
> 
> >Oi pessoal,
> >Como faço para obter senos e cossenos em radicais reais? Por exemple sen36?
> >obrigado
> >abraços
> >marcelo

Para calcular senx para x=36graus, um processo trigonometrico eh
observar que sen3x=sen2x. Chega-se a uma equaçao que permite o calculo
de senx.



Re: Grafos

2000-10-20 Por tôpico Augusto Morgado



Eduardo Favarão Botelho wrote:
> 
> Olá pessoal...
> 
> Li o artigo sobre circuitos eulerianos no primeira Eureka, e lá era
> enunciado o seguinte teorema:
> "Existe um circuito euleriano em um grafo se e somente se o grafo é conexo e
> cada vértice tem  grau par".
> 
> Depois de ler o teorema, pensei num quadrado ABCD com uma bissetriz
> traçada, AC por exemplo.
> Ora, o circuito não tem grau par, pois nos vértices A e C incidem três
> arcos, número ímpar.
> O que há de errado?
> 
> Abraços, Eduardo.
Um circuito começa e termina no mesmo ponto, No seu grafo realmente não
existe um circuito euleriano. Não há nada de errado.
O que existe é um caminho aberto (começa e termina em pontos diferentes)
euleriano. A condição de existência de um caminho aberto euleriano é
similar, só que deve haver dois vértices de grau ímpar, os quais serão
precisamente os pontos inicial e final do caminho.



Re: Sobre as =?x-user-defined?q?fun=E7=F5es=20trigonom=E9tricas?= de =?x-user-defined?q?n=FAmeros?= complexos

2000-10-17 Por tôpico Augusto Morgado



> Jorge Peixoto Morais wrote:
> 
> Eu estava procurando uma maneira de definir  funcoes trigonometricas
> de numeros complexos, e lembrei a formula de Euler (exp(ix)= cos(x) +
> i*sen(x)). Sera que entao
> cos(i) + i*sen(i)= exp(i*i)=exp(-1)= 1/e ? Eu ficaria feliz so por
> conseguir a resposta dessa ultima pergunta. Mas, se tambem der para
> explicar mais sobre seno e cosseno de numeros complexos, melhor.
>  
> PS: A*B denota "A vezes B".
Eh exatamente isso.As funçoes seno e co-seno sao definjdas nos complexos
por cosz +isenx=expz e cosz-isenz=exp(-z).



Re: "Jigsaws puzzles"

2000-10-16 Por tôpico Augusto Morgado



> Hugo Iver Vasconcelos Goncalves wrote:
> 
> Qual é a explicação matemática para os chamados "Jigsaws Puzzles" (em
> especial aquele do triangulo dividido em quatro partes onde quando as
> rearanjamos um buraco aparece) ? Qual a relação dela com os números de
> Fibonacci?
Embora não seja um especialista nesse tipo de puzzles, o que está
circulando a internet (o do triângulo) tem uma explicação muito simples.
Certos pontos no primeiro desenho não estão alinhados e por sutilissimas
deformações de escala sao desenhados como colineares. No segundo
desenho, a mesma coisa, forçando ao contrário é claro, para que apareça
o buraco.
Morgado



Re: =?x-user-defined?q?Recorr=EAncia?=

2000-10-06 Por tôpico Augusto Morgado



Marcelo Souza wrote:
> 
> Olá pessoal! Alguém poderia me explicar detalhadamente o exercício a seguir?
> 
> . Defina, por recorrência, uma função f:N->N estipulando que f(1)=3 e
> f(n+1)=5.f(1)+1. Dê uma forma explícita para f(n).
> 
> Obrigado
> Abraços
> Marcelo
NAO SERIA f(n+1)=5.f(n)+1?



Re: =?x-user-defined?q?coment=E1rios?=

2000-09-30 Por tôpico Augusto Morgado



josimat wrote:
> 
> Nesta versão, o espaço amostral fica reduzido às duas situações nas quais o
> juíz vê a face vermelha:   vermelha/vermelha e vermelha/amarela.
> Dessas duas, apenas uma verifica o enunciado. Logo, a resposta é 1/2.
> 
> ALGUÉM PODERIA ME AJUDAR COM ESTE PROBLEMA? Já o coloquei nesta lista, numa
> mensagem, juntamente com outros, mas talvez ninguém tivesse reparado.
> 
> Arremessa-se um dado até que se obtenha o número 5 (uma vez obtido o número
> 5, cessam-se os arremessos). Qual a probabilidade de obtermos só um número 4
> entre o quinto e o oitavo lançamento?
> 
> []'s JOSIMAR
> 
> -Mensagem original-
> De: Nicolau C. Saldanha <[EMAIL PROTECTED]>
> Para: [EMAIL PROTECTED] <[EMAIL PROTECTED]>
> Data: Sábado, 30 de Setembro de 2000 08:09
> Assunto: Re: comentários
> 
> >
> >
> >On Sat, 30 Sep 2000, Alexandre F. Terezan wrote:
> >
> >> Olá,
> >>
> >> Aparentemente a resposta é simples.  Para q o enunciado ocorra,
> primeiramente
> >> o juiz deverá escolher o cartao bicolor (probabilidade de 1/3) e, além
> disso,
> >> este cartao deverá ter a sua cor vermelha voltada para o juiz (1/2 de
> >> probabilidade)
> >>
> >> Assim, a probabilidade geral é de 1/2 * 1/3 = 1/6.
> >>
> >>
> >>   Um juiz de futebol possui três cartões no bolso. Um é todo amarelo,
> outro é
> >>   todo vermelho e o terceiro é vermelho de um lado e amarelo do outro.
> Num
> >>   determinado lance, o juiz retira, ao acaso, um cartão do bolso e mostra
> a
> >>   um jogador. Determine a probabilidade da face que o juiz vê ser
> vermelha e
> >>   da outra face mostrada ao jogador ser amarela.
> >>
> >
> >O Alexandre tem razão, claro.  Uma variante mais interessante seria igual,
> >exceto pela última frase, que fica assim:
> >
> >Determine a probabilidade de que a face mostrada ao jogador seja amarela
> >dado que a face que o juiz vê é vermelha.
> >
> >Serão essas duas situações igualmente prováveis?



Re: Mas que vergonha...

2000-09-23 Por tôpico Augusto Morgado



 On Thu, 21 Sep 2000, Jorge Peixoto  Morais wrote:
Eu falei uma grande besteira na mensagem em que respondi o Villard... É
que eu ainda estou na oitava e não estudei essas coisas. Eu comecei a
pouco tempo a estudar física além da matéria da escola, e por isso ainda
sabia muito pouco além da matéria da oitava... Então logo depois de
perguntar se o fato  era devido à deformação da bola de basquete eu vi
no livro que se não
houvesse deformação o corpo nem ricochetearia (o que, aliás, é lógico).
Mas eu já havia escrito besteira. Mas que vergonha...

> Não é vergonha nenhuma errar.
> 
> Quanto ao que ocorreria "se não houvesse deformação",
> bem, ocorreria uma violação muito fundamental das leis da física:
> qualquer objeto sólido deforma, uns mais, uns menos, mas todos deformam.
> Considere  que quando a parte de baixo da bola bate no chão a parte de cima
> ainda não "sabe" que deve parar de cair e só pode receber o recado
> na melhor das hipóteses na velocidade da luz:
> durante a mínima fração de segundo entre o sinal viajar
> entre um lado da bola e o outro,
> a parte de baixo da bola já parou de cair mas a de cima continua caindo
> e a bola portanto se deforma.
> Mas este argumento fundamental provavelmente subestima a deformação real
> por várias ordens de grandeza. Lembre que um objeto sólido é uma nuvem
> de núcleos atômicos e elétrons que se mantem juntos por forças eletromagnéticas
> (e outras).
> 
> []s, N.

Fica frio, Jorge. Eu, por exemplo, já escrevi nesta lista besteiras
muito (bota muito nisso) piores. Todos nós sabemos andar, mas às vezes
tropeçamos.
 Morgado



Re: Duvida

2000-09-22 Por tôpico Augusto Morgado



David Pereira wrote:
> 
> Como se calcula a distancia entre retas no R3??? E como isso pode ser
> provado?
> 
> []s
> David
Tome dois pontos A e B na primeira reta e dois pontos C e D na segunda
reta. O produto vetorial de AB por CD da como resultado um vetor V que
tem a direçao da perpendicular as duas retas. A distancia procurada eh
simplesmente o modulo da projeçao de AC sobre V, ou seja, o modulo do
produto esalar de AC por V, dividido pelo modulo de V.



Re: =?x-user-defined?q?combinat=F3ria=2Dajuda?=

2000-09-19 Por tôpico Augusto Morgado



> Filho wrote:
> 
> Quantos anagramas podemos formar com as letras da palavra AMOR de modo
> que n?o apare?am: na primeira posi??o, a letra A; na segunda, a letra
> M; na terceira, a letra O e na quarta, a letra R ?
A resposta é o numero de permutaçoes caoticas de 4 elementos, que vale
4!(1/0! - 1/1! +1/2!-1/3!+1/4!)= 9.
Leia o Análise Combinatoria e Probabilidade publicado pela SBM na
Coleçao do Professor de Matematica.
Morgado



Re: PA

2000-09-15 Por tôpico Augusto Morgado



Fabricio Damasceno wrote:
> 
> Em Thu, 10 Aug 2000 23:14:38 -0300 Augusto Morgado Escreveu:
> 
> >
> >
> > > Aron Roberto Ferreira wrote:
> > >
> > > Ol?!   
> > >     Tentei resolver o problema abaixo e a resposta nao bate com a do
> > > autor(Bezerra). Alguem da lista poderia conferir minha resolucao?
> > > O problema:
> > > "Qual e a razao de uma progressao aritmetica em que a*1 = 1 e a*
> (n+2)
> > > = n^2".
> 
> > > Resolucao
> > > escrevendo a PA (1, ...,n^2,...)
> > > pelo termo geral eu fiz
> > > n^2=1+(n-1)r AQUI ESTA O ERRO, ESTE EH O TERMO DE ORDEM n+2 E NAO DE
> > ORDEM n. DEVERIA SER n^2=1+((n+2)-1)r
> > > n^2-1= (n-1)r
> > > (n^2-1)/(n-1)=r
> > > (n+1)(n-1)/(n-1)=r
> > > r= n+1
> > > A resposta do autor e (n - 1).
> > > ( a*1 significa a indice 1 )
> > >     Se eu errei, me ajudem.
> > > obrigado!
> ---
>   Analise desta forma:
>  Veja os dados:
> a*1=1
> a*(n+2)= n^2
> 
> colocando n+2=t ,temos
> a*t=(t-2)^2 , o que nos daria a*t=t^2 -4t +t
> considerando que essa ultima expressao e a formula para o termo geral(NAO EH NAO. n 
>NESTE PROBLEMA NAO EH UMA VARIAVEL. EH UMA CONSTANTE) e
> que somente queremos calcular o valor da razao. Pq nao substituimos os
> valores de t (t pertencente aos naturais diferentes de zero)e
> calculamos a razao, visto que isso nos forneceria um valor independente
> de t (constante). Pq o valor de a*1=1 a*2=0 e a*3=1. Essa ultima
> observacao nao descaracterizaria a PA.
> 
> Abracos!!
> 
> MailBR - O e-mail do Brasil -- http://www.mailbr.com.br
> Faça já o seu. É gratuito!!!



Re: =?x-user-defined?q?Permuta=E7=F5es=20ca=F3ticas?=

2000-09-02 Por tôpico Augusto Morgado



"Nicolau C. Saldanha" wrote:
> 
> On Sat, 2 Sep 2000, Franklin de Lima Marquezino wrote:
> 
> > Olá,
> >
> > Há algum tempo eu perguntei nesta lista, como se calcula permutações
> > caóticas, e ninguém me respondeu até hoje. Estou no 3º ano do ensino
> > médio, e sei que minha pergunta pode ter sido um pouco idiota. Porém, eu
> > já havia comentado esta minha dúvida com alguns de meus amigos, que
> > também participam desta lista, e nenhum deles soube responder. Então,
> > como vocês dizem que aqui predomina a solidariedade e a cooperação, eu
> > pensei que não houvesse problema em compartilhar minha dúvida, por mais
> > simples que fosse.  Caso alguém tenha tempo, por favor, responda.
> >
> >
> >Até logo,
> >
> >
> >  Franklin
> >
> >
> 
> Sua pergunta não tem nada de idiota. Não sei mais se você foi a pessoa
> que perguntou sobre os anagramas da palavra MATEMATICA onde não há
> nenhuma coincidência de letra em nenhuma das posições:
> esta pergunta eu pensei um pouco sobre ela e só não respondi por que
> achei trabalhosa (apesar de ser interessante).
> Mas se você está perguntando sobre permutações caóticas na situação
> simples, nenhum elemento repetido, a resposta é:
> 
> n!(1/0! - 1/1! + 1/2! - 1/3! + ... + (-1)^n 1/n!)
> 
> que é o inteiro mais próximo de n!/e.
> A demonstração é feita pelo princípio da inclusão e exclusão
> e só não repito aqui agora pq acho que alguém já tem isso pronto
> escrito em algum lugar. Se ninguém der referência ou se a referência
> não for acessível eu demonstro aqui.
> 
> []s, N.
Tá no "Análise Combinatória e Probabilidade", de Morgado, Pitombeira,
Fernandez e Paulo Cezar Carvalho.
Morgado



Re: Dia da semana

2000-08-23 Por tôpico Augusto Morgado



Fernando Lucchesi wrote:
> 
> Caro Augusto,
> não entendi por que que os anos múltiplos de 100 não são bissextos. No meu
> entendimento, anos bissextos acontecem de 4 em 4 anos, e sendo 2000, um ano
> bissexto, todos os múltiplos de 4 seriam. Gostaria, se fosse possivel, de
> maiores esclarecimentos.
>  Fernando Lucchesi

Nos tempos de Júlio César (instituidor do calendario juliano que foi
usado ate cerca de 1600) pensava-se (hoje se sabe que isso so eh verdade
aproximadamente) que o ano (periodo que a Terra demora para dar uma
volta em torno do Sol) era igual a 365 e 1/4 dias (dia=periodo que a
Terra demora para dar uma volta em torno de si mesma). No calendario
juliano, de quatro em quatro anos aparecia um bissexto. Em torno de 1560
os problemas ja haviam surgido. As estacoes do ano nao estavam comecando
nos dias em que deveriam, o que causava enormes transtornos a
agricultura. O papa entao estabeleceu o calendario gregoriano para
acertar as estacoes. Entao foi estabelecida essa regra dos anos
terminados em 00 (isso garante as estacoes comecando nos dias certos ate
3200 ou 6400, nao me lembro agora; provavelmente quando la chegarmos se
estabelecerah que os anos multiplos de 3200(ou serah 6400?), que pelas
regras atuais sao bissextos, deixaram de se-lo; mas ate la estaremos
todos mortos. Quando foi instituido o calendario gregoriano, houve uma
correcao dos erros acumulados anteriormente e a folhinhas deu um pulo de
15 dias. Se plo calendario atual voce viajar para o passado dia a dia e
for a Porto Seguro, voce vai ver Cabral aqui chegando no inicio de maio
e se perguntar aos portugueses que dia eh, eles dirao ser 22 de abril,
pois estao com o calendario juliano.
Veja as mensagens mandadas por josimat e antonio neto.
Morgado



Re: Dia da semana

2000-08-22 Por tôpico Augusto Morgado



Ecass Dodebel wrote:
> 
> >From: Augusto Morgado <[EMAIL PROTECTED]>
> >Reply-To: [EMAIL PROTECTED]
> >To: [EMAIL PROTECTED]
> >Subject: Re: Dia da semana
> >Date: Mon, 21 Aug 2000 10:38:47 -0300
> >
> >
> >
> >Ecass Dodebel wrote:
> > >
> > > >From: "Wellington Ribeiro de Assis" <[EMAIL PROTECTED]>
> > > >Reply-To: [EMAIL PROTECTED]
> > > >To: "discusspio de problemas" <[EMAIL PROTECTED]>
> > > >Subject: Dia da semana
> > > >Date: Fri, 18 Aug 2000 23:59:17 -2:00
> > > >
> > > >Prezados amigos
> > > >
> > > >Alguem sabe dizer como eh o algoritmo usado para se descobrir que dia
> > > >da semana cai uma determinada data de um ano qualquer?
> > > >
> > > >Bons estudos e abraco a todos,
> > > >Wellington
> > >
> > > Olá,
> > >
> > > eu tenho uma idéia. Conseguiremos descobrir em que dia da semana cai o
> >dia D
> > > do mês M do ano A, se soubermos que dia da semana foi 1/1/1 (por
> >exemplo), e
> > > quantos dias já se passaram até o dia D/M/A desde 1/1/1, e fazer o resto
> >da
> > > divisão por 7, o resto 0 nos dirá que D/M/A é o mesmo dia da semana de
> > > 1/1/1, o resto 1 que D/M/A é um dia depois, e assim por diante. Vou
> > > apresentar uma possível solução. Defino o seguinte:
> > > - DA(A) = número de dias dos anos entre o ano 1 e A-1 (inclusive), para
> >A>0,
> > > não vou considerar os casos com o ano negativo.
> > > - DM(M) = número de dias dos meses entre 1 e M-1, do ano A.
> > > - DD(D) = número de dias anteriores a D, no mês M do ano A.
> > > O resto procurado é o da divisão de
> >ND(D/M/A)=DA(A)+DM(M)+DD(D)-ND(1/1/1)
> > > por 7.
> > > Eu não vou falar muitos detalhes (para não ficar muito chato). Tomando a
> > > função [x], menor inteiro, que diz o natural N, tal que N<=x > > [x]=N, temos
> > > DA(A) = 365(A-2) + [(A-1)/4], se A>0
> > > DM(M) = 31([M/2] + [M/9] - [M/10] + [M/11] - [M/12])
> > > + 30([M/5] + [M/7] + [M/12])
> > > + 28([M/3] - [M/6] - [M/9] - [M/12])
> > > + [1 - A/4 + [A/4]]([M/3] - [M/6] - [M/9] - [M/12])
> > > DD(D) = D
> > > Agora podemos fazer algumas simplicações módulo 7.
> > >
> > > ND(D/M/A) =
> > >
> 
>>(A-2)+[(A-1)/4]+3([M/2]+[M/9]-[M/10]+[M/11]-[M/12])+2([M/5]+[M/7]+[M/12])+[1-A/4+[A/4]]([M/3]-[M/6]-[M/9]-[M/12])+D
> > > (mod 7)
> > >
> > > Hoje, ND(20/8/2000) = 0 (mod 7), domingo.
> > > Primeiro dia da era cristã, ND(1/1/1) = 0 (mod 7), domingo.
> > > Proclamação da Independência, ND(7/9/1922) = 5 (mod 7), sexta-feira.
> > > Proclamação da República, ND(15/11/1889) = 4 (mod 7), quinta-feira.
> > >
> > > Eu tenho quase certeza de que para anos A negativos, uma expressão
> >válida é
> > > a seguinte:
> > >
> > > ND(D/M/A) =
> > >
> 
>>A-[(|A|+3)/4]+3([M/2]+[M/9]-[M/10]+[M/11]-[M/12])+2([M/5]+[M/7]+[M/12])+[1-(A+1)/4+[(A+1)/4]]([M/3]-[M/6]-[M/9]-[M/12])+D
> > > (mod 7),
> > >
> > > ela deixa os restos similares aos da expressão com o A>0, ou seja, 0
> > > continua sendo domingo, e assim por diante.
> > >
> > > Eu não uni os dois casos (A>0 e A<0) numa expressão só, por que não
> >existe o
> > > ano 0, e isso complicou tudo para mim, talvez eu até conseguisse, mas
> > > ficaria algo tão comprido que prefiro nem tentar.
> > >
> > > Obrigado!
> > >
> > > Eduardo Casagrande Stabel.
> > >
> >Eduardo:
> >Nao examinei com cuidado o que voce fez. Mas a primeira impressao eh que
> >nao esta correto porque me parece que voce esta considerando que todos
> >os anos multiplos de 4 sejam bissextos, o que nao eh verdade. Em suma,
> >parece que a matematica eh boa mas os anos bissextos infelizmente nao
> >sao os multiplos de 4.
> >
> >Anos bissextos sao todos os que sao multiplos de 4 sem ser de 100; os
> >multiplos de 100 so sao bissextos se forem multiplos de 400.
> >Morgado
> 
> Olá prof. Morgado,
> eu não sabia desse dado sobre os anos bissextos, pensei, como você bem
> citou, que os múltiplos de 4 eram bissextos. Bom, com o dado que você passa,
> acho que não fico muito difícil de corrigir a minha fórmula, mas vou pensar
> melhor. Tenho uma dúvida: e quanto aos anos AC? como ficam os bissextos?
> Obrigado pela correção!
> 
> Eduardo Casagrande Stabel.

Nao ha, na minha opiniao, muito sentido nos anos AC porque este nosso
calendario so foi adotado nos paises ocidentais em torno de 1570
(1567? ou 1583, sei la).Quando foi adotado o calendario nosso
(gregoriano) em substituicao ao juliano, houve uma correcao e varios
dias nao existiram. Se fossemos retrocedendo pelo nosso calendario dia a
dia encontrariamos Cabral chegando ao Brasil em maio e nao em 22 de
abril de 1500. 
Na Russia, o calendario gregoriano so foi adotado neste seculo XX; basta
ver que a revolucao que eles chamam de outubro ocorreu pelo nosso
calendario em novembro.
Procure ver um artigo meu na RPM sobre isso.
Morgado



Re: Dia da semana

2000-08-21 Por tôpico Augusto Morgado



Ecass Dodebel wrote:
> 
> >From: "Wellington Ribeiro de Assis" <[EMAIL PROTECTED]>
> >Reply-To: [EMAIL PROTECTED]
> >To: "discusspio de problemas" <[EMAIL PROTECTED]>
> >Subject: Dia da semana
> >Date: Fri, 18 Aug 2000 23:59:17 -2:00
> >
> >Prezados amigos
> >
> >Alguem sabe dizer como eh o algoritmo usado para se descobrir que dia
> >da semana cai uma determinada data de um ano qualquer?
> >
> >Bons estudos e abraco a todos,
> >Wellington
> 
> Olá,
> 
> eu tenho uma idéia. Conseguiremos descobrir em que dia da semana cai o dia D
> do mês M do ano A, se soubermos que dia da semana foi 1/1/1 (por exemplo), e
> quantos dias já se passaram até o dia D/M/A desde 1/1/1, e fazer o resto da
> divisão por 7, o resto 0 nos dirá que D/M/A é o mesmo dia da semana de
> 1/1/1, o resto 1 que D/M/A é um dia depois, e assim por diante. Vou
> apresentar uma possível solução. Defino o seguinte:
> - DA(A) = número de dias dos anos entre o ano 1 e A-1 (inclusive), para A>0,
> não vou considerar os casos com o ano negativo.
> - DM(M) = número de dias dos meses entre 1 e M-1, do ano A.
> - DD(D) = número de dias anteriores a D, no mês M do ano A.
> O resto procurado é o da divisão de ND(D/M/A)=DA(A)+DM(M)+DD(D)-ND(1/1/1)
> por 7.
> Eu não vou falar muitos detalhes (para não ficar muito chato). Tomando a
> função [x], menor inteiro, que diz o natural N, tal que N<=x [x]=N, temos
> DA(A) = 365(A-2) + [(A-1)/4], se A>0
> DM(M) = 31([M/2] + [M/9] - [M/10] + [M/11] - [M/12])
> + 30([M/5] + [M/7] + [M/12])
> + 28([M/3] - [M/6] - [M/9] - [M/12])
> + [1 - A/4 + [A/4]]([M/3] - [M/6] - [M/9] - [M/12])
> DD(D) = D
> Agora podemos fazer algumas simplicações módulo 7.
> 
> ND(D/M/A) =
> 
>(A-2)+[(A-1)/4]+3([M/2]+[M/9]-[M/10]+[M/11]-[M/12])+2([M/5]+[M/7]+[M/12])+[1-A/4+[A/4]]([M/3]-[M/6]-[M/9]-[M/12])+D
> (mod 7)
> 
> Hoje, ND(20/8/2000) = 0 (mod 7), domingo.
> Primeiro dia da era cristã, ND(1/1/1) = 0 (mod 7), domingo.
> Proclamação da Independência, ND(7/9/1922) = 5 (mod 7), sexta-feira.
> Proclamação da República, ND(15/11/1889) = 4 (mod 7), quinta-feira.
> 
> Eu tenho quase certeza de que para anos A negativos, uma expressão válida é
> a seguinte:
> 
> ND(D/M/A) =
> 
>A-[(|A|+3)/4]+3([M/2]+[M/9]-[M/10]+[M/11]-[M/12])+2([M/5]+[M/7]+[M/12])+[1-(A+1)/4+[(A+1)/4]]([M/3]-[M/6]-[M/9]-[M/12])+D
> (mod 7),
> 
> ela deixa os restos similares aos da expressão com o A>0, ou seja, 0
> continua sendo domingo, e assim por diante.
> 
> Eu não uni os dois casos (A>0 e A<0) numa expressão só, por que não existe o
> ano 0, e isso complicou tudo para mim, talvez eu até conseguisse, mas
> ficaria algo tão comprido que prefiro nem tentar.
> 
> Obrigado!
> 
> Eduardo Casagrande Stabel.
> 
Eduardo:
Nao examinei com cuidado o que voce fez. Mas a primeira impressao eh que
nao esta correto porque me parece que voce esta considerando que todos
os anos multiplos de 4 sejam bissextos, o que nao eh verdade. Em suma,
parece que a matematica eh boa mas os anos bissextos infelizmente nao
sao os multiplos de 4.

Anos bissextos sao todos os que sao multiplos de 4 sem ser de 100; os
multiplos de 100 so sao bissextos se forem multiplos de 400.
Morgado



Re: Dia da semana

2000-08-19 Por tôpico Augusto Morgado



Wellington Ribeiro de Assis wrote:
> 
> Prezados amigos
> 
> Alguem sabe dizer como eh o algoritmo usado para se descobrir que dia
> da semana cai uma determinada data de um ano qualquer?
> 
> Bons estudos e abraco a todos,
> Wellington
A RPM publicou um artigo, de A. C. Morgado (ou seja, eu), provando a
fórmula que permite tal calculo.
Um abraco.
Morgado



Re: problemas

2000-08-11 Por tôpico Augusto Morgado



Augusto Morgado wrote:
> 
> Augusto Morgado wrote:
> >
> > Marcelo Souza wrote:
> > >
> > > Fala, galera da lista,
> > > Estou com 2 problemas que estou precisando da solução o mais rápido
> > > possível. São eles:
> > > 1. Calcular o menor número natural n tal que:
> > > a) O último algarismo na sua representação decimal é 6.
> > > b) Se apagarmos o último algarismo 6 e colocarmos ele na frente de todos os
> > > outros dígitos, o novo número formado será quatro vezes o anterior.
> > >
> > > 2. Os números reais a e b, cumprem
> > >
> > > a^3 - 3a^2 + 5a - 17 = 0
> > > b^3 - 3b^2 + 5b + 11 = 0, calcule a + b
> > >
> > > Espero respostas
> > > Obrigado
> > > Abraços
> > > Marcelo
> > 2) Chame a soma a+b de S. Substitua b por S-a e some as equacoes. Apos
> > muitas contas encontra-se 3(S-2)a^2-3S(S-2)a+ (S-2)(S^2-s+3)=0.
> > S=2 eh uma solucao. As outras sao obtidas dividindo o polinomio acima
> > por S-2.
> > Dividindo encontra-se 3a^2-3Sa+S^2-S+3=0. Esta equacao em a so tera
> > solucao real se for 9S^2 >= 12(S^2-S+3).
> > Simplificando obtemos S^2 -4S +12 >=0 ~(ERRO DE DIGITACAO)e esta condicao eh 
>impossivel de
> > ser satisfeita por S real. Logo, a unica solucao eh S=2.
> > Espero nao ter errado conta nenhuma!
> > Morgado
> PS: Nao errei conta, mas errei a digitacao. O ultimo sinal de >= eh na
> verdade um sinal de <(ERREI DE NOVO) ESSA DROGA EH UM SINAL DE MENOR OU IGUAL



Re: problemas

2000-08-11 Por tôpico Augusto Morgado



Marcelo Souza wrote:
> 
> Fala, galera da lista,
> Estou com 2 problemas que estou precisando da solução o mais rápido
> possível. São eles:
> 1. Calcular o menor número natural n tal que:
> a) O último algarismo na sua representação decimal é 6.
> b) Se apagarmos o último algarismo 6 e colocarmos ele na frente de todos os
> outros dígitos, o novo número formado será quatro vezes o anterior.
> 
> 2. Os números reais a e b, cumprem
> 
> a^3 - 3a^2 + 5a - 17 = 0
> b^3 - 3b^2 + 5b + 11 = 0, calcule a + b
> 
> Espero respostas
> Obrigado
> Abraços
> Marcelo
1) 615384
Eh meio dificil explicar, mas a ideia eh pensar no numero como

- - - - - - 6
x4
__

e ir multiplicando. Eh claro que o produto termina em 4, logo 4 eh o
penultimo algarismo do multiplicando, e assim vai.
Morgado



Re: problemas

2000-08-11 Por tôpico Augusto Morgado



Augusto Morgado wrote:
> 
> Marcelo Souza wrote:
> >
> > Fala, galera da lista,
> > Estou com 2 problemas que estou precisando da solução o mais rápido
> > possível. São eles:
> > 1. Calcular o menor número natural n tal que:
> > a) O último algarismo na sua representação decimal é 6.
> > b) Se apagarmos o último algarismo 6 e colocarmos ele na frente de todos os
> > outros dígitos, o novo número formado será quatro vezes o anterior.
> >
> > 2. Os números reais a e b, cumprem
> >
> > a^3 - 3a^2 + 5a - 17 = 0
> > b^3 - 3b^2 + 5b + 11 = 0, calcule a + b
> >
> > Espero respostas
> > Obrigado
> > Abraços
> > Marcelo
> 2) Chame a soma a+b de S. Substitua b por S-a e some as equacoes. Apos
> muitas contas encontra-se 3(S-2)a^2-3S(S-2)a+ (S-2)(S^2-s+3)=0.
> S=2 eh uma solucao. As outras sao obtidas dividindo o polinomio acima
> por S-2.
> Dividindo encontra-se 3a^2-3Sa+S^2-S+3=0. Esta equacao em a so tera
> solucao real se for 9S^2 >= 12(S^2-S+3).
> Simplificando obtemos S^2 -4S +12 >=0 ~(ERRO DE DIGITACAO)e esta condicao eh 
>impossivel de
> ser satisfeita por S real. Logo, a unica solucao eh S=2.
> Espero nao ter errado conta nenhuma!
> Morgado
PS: Nao errei conta, mas errei a digitacao. O ultimo sinal de >= eh na
verdade um sinal de <



Re: problemas

2000-08-11 Por tôpico Augusto Morgado



Marcelo Souza wrote:
> 
> Fala, galera da lista,
> Estou com 2 problemas que estou precisando da solução o mais rápido
> possível. São eles:
> 1. Calcular o menor número natural n tal que:
> a) O último algarismo na sua representação decimal é 6.
> b) Se apagarmos o último algarismo 6 e colocarmos ele na frente de todos os
> outros dígitos, o novo número formado será quatro vezes o anterior.
> 
> 2. Os números reais a e b, cumprem
> 
> a^3 - 3a^2 + 5a - 17 = 0
> b^3 - 3b^2 + 5b + 11 = 0, calcule a + b
> 
> Espero respostas
> Obrigado
> Abraços
> Marcelo
2) Chame a soma a+b de S. Substitua b por S-a e some as equacoes. Apos
muitas contas encontra-se 3(S-2)a^2-3S(S-2)a+ (S-2)(S^2-s+3)=0.
S=2 eh uma solucao. As outras sao obtidas dividindo o polinomio acima
por S-2.
Dividindo encontra-se 3a^2-3Sa+S^2-S+3=0. Esta equacao em a so tera
solucao real se for 9S^2 >= 12(S^2-S+3). 
Simplificando obtemos S^2 -4S +12 >=0 e esta condicao eh impossivel de
ser satisfeita por S real. Logo, a unica solucao eh S=2.
Espero nao ter errado conta nenhuma!
Morgado



Re: PA

2000-08-10 Por tôpico Augusto Morgado



> Aron Roberto Ferreira wrote:
> 
> Ol?!   
>     Tentei resolver o problema abaixo e a resposta nao bate com a do
> autor(Bezerra). Alguem da lista poderia conferir minha resolucao?
> O problema:
> "Qual e a razao de uma progressao aritmetica em que a*1 = 1 e a*(n+2)
> = n^2".
> Resolucao
> escrevendo a PA (1, ...,n^2,...)
> pelo termo geral eu fiz
> n^2=1+(n-1)r AQUI ESTA O ERRO, ESTE EH O TERMO DE ORDEM n+2 E NAO DE ORDEM n. 
>DEVERIA SER n^2=1+((n+2)-1)r
> n^2-1= (n-1)r
> (n^2-1)/(n-1)=r
> (n+1)(n-1)/(n-1)=r
> r= n+1
> A resposta do autor e (n - 1).
> ( a*1 significa a indice 1 )
>     Se eu errei, me ajudem.
> obrigado!



Re: Problema de Geometria Plana

2000-08-06 Por tôpico Augusto Morgado



Ponce wrote:
> 
> Caros amigos e ao meu grande amigo Carlos Victor,
> 
> A solu??o dada pelo Carlos Victor ? prefeita, para o caso
> do quadril?tero ABCD ser convexo, o que n?o foi dito no enunciado.!!.
> Entretanto, o resultado ? verdadeiro mesmo no caso de n?o ser convexo,
> e isso deixo para voc?s constaterem a veracidade.
> Acredito ainda que seria interessante limitarmos inferiormente o valor
> de MN.
> Um abra?o
> PONCE
>  
>  
> 
> Carlos Victor wrote:
> 
> > At 23:43 04/08/00 -0300, josimat wrote:
> > >A? vai o problema da prova do Col?gio Naval (?ltima ter?a feira):
> > >
> > >Seja ABCD um quadril?tero qualquer onde os lados opostos N?O s?o
> > paralelos.
> > >Se as medidas dos lados opostos AB e DC s?o, respectivamente,
> > iguais a 12 e
> > >16, um valor poss?vel para o segmento de extremos M (ponto m?dio do
> > lado AD)
> > >e N (ponto m?dio do lado BC) ?
> > >(A) 12,5   (B) 14    (C) 14,5
> > >(D) 16   (E) 17
> > >
> > >JOSIMAR
> > >-Mensagem original-
> > >De: Augusto Morgado <[EMAIL PROTECTED]>
> > >Para: [EMAIL PROTECTED] <[EMAIL PROTECTED]>
> > >Data: Sexta-feira, 4 de Agosto de 2000 03:03
> > >Assunto: Re: Problema de Geometria Plana
> > >
> > >
> > > >
> > > >
> > > >Eduardo Quintas da Silva wrote:
> > > >>
> > > >> Seja ABCD um quadril?tero onde os lados opostos n?o s?o
> > paralelos, AB =
> > > >> 12, CD = 16, M ponto m?dio de AD e N ponto m?dio de
> > BC
> > > >> calcule MN.
> > > >Eduardo:
> > > >O problema nao admite resposta, esta super indeterminado.
> > > >Parece que a intencao eh o problema que caiu na prova do colegio
> > naval.
> > > >Se for, o fato e que se fosse um trapezio a resposta seria 14.
> > Nao
> > > >sendo, porque os lados nao sao paralelos, a resposta certa na
> > prova era
> > > >que MN tinha que ser menoe que 14.
> > > >
> > > >Sugiro que voce jogue de novo o problema na lista, mas completo.
> > > >Morgado
> >
> > Nota : "A quest?o  pedia um poss?vel valor  para  MN " .
> > Para este  problema, fa?a  o seguinte : Trace a diagonal AC e seja P
> > o
> > ponto  m?dio de  AC . Como AB  e CD n?o s?o paralelos  , temos
> > formado o
> > tri?ngulo MNP. Como NP = 6   e MP =  8 e pela exist?ncia de MNP ,
> > conclu?mos que  MN <  14 ; das op??es a ?nica  poss'vel ? 12,5  ou
> > seja ,
> > op??o  A , ok ?
> >
> > Abra?os , Carlos  Victor

Seja ABCD o quadrilatero convexo ou nao. Sejam M medio de AD e N medio
de BC. Usemos vetores. MN = N-M = (B+C)/2 - (A+D)/2 = 1/2 (B-A+C-D)=
1/2(AB+DC). Como o modulo da soma esta compreendido entre a soma e a
diferença dos modulos, o comprimento de MN esta entre 14 e 2.
Morgado



Re: Problema de Geometria Plana

2000-08-04 Por tôpico Augusto Morgado



Eduardo Quintas da Silva wrote:
> 
> Obrigado Prof. Morgado... Trata-se sim do problema proposto na prova do
> colégio naval não tenho o enunciado completo da questão... escrevi da
> maneira que me foi proposta
> 
> Valeu !
> 
> Augusto Morgado escreveu:
> 
> > Eduardo Quintas da Silva wrote:
> > >
> > > Seja ABCD um quadrilátero onde os lados opostos não são paralelos, AB =
> > > 12, CD = 16, M ponto médio de AD e N ponto médio de BC
> > > calcule MN.
> > Eduardo:
> > O problema nao admite resposta, esta super indeterminado.
> > Parece que a intencao eh o problema que caiu na prova do colegio naval.
> > Se for, o fato e que se fosse um trapezio a resposta seria 14. Nao
> > sendo, porque os lados nao sao paralelos, a resposta certa na prova era
> > que MN tinha que ser menoe que 14.
> >
> > Sugiro que voce jogue de novo o problema na lista, mas completo.
> > Morgado

Vou tentar descrever o problema da prova do colegio naval, claro que nao
de modo textual pois nao tenho a prova.

Em um quadrilatero nao ha lados paralelos e dois lados opostos AB e CD
medem 12 e 16. O segmento que une os pontos medios de BC e DA PODE
MEDIR:
Ai surgem as cinco alternativas. Uma delas vale 14, outra eh 12,5 e as
tres outras sao maiores que 14.

Nao eh dificil provar, e este fica sendo o problema para a lista, que o
fato de nao haver lados paralelos implica que o segmento que une os
pontos medios deve ser menor que a media entre 12 e 16, que eh 14.
Morgado



Re: Problema de Geometria Plana

2000-08-03 Por tôpico Augusto Morgado



Eduardo Quintas da Silva wrote:
> 
> Seja ABCD um quadrilátero onde os lados opostos não são paralelos, AB =
> 12, CD = 16, M ponto médio de AD e N ponto médio de BC
> calcule MN.
Eduardo:
O problema nao admite resposta, esta super indeterminado.
Parece que a intencao eh o problema que caiu na prova do colegio naval.
Se for, o fato e que se fosse um trapezio a resposta seria 14. Nao
sendo, porque os lados nao sao paralelos, a resposta certa na prova era
que MN tinha que ser menoe que 14.

Sugiro que voce jogue de novo o problema na lista, mas completo.
Morgado



Re: Pergunta solta

2000-07-30 Por tôpico Augusto Morgado

 Caro Morgado,
 eu agradeço a sua brilhante solução e ressalto que na primeira parte
dela,
 você usa a área de uma curva para estimar a soma de parcelas. É uma
idéia
 bem simples, mas eu nunca havia pensado nisso. Parece ser muito útil
fazer
 esse tipo de estimativas:
 Digamos que queremos estimar 1/1+1/2+...+1/k.
 Seguindo o raciocínio do prof. e usando notação similar. Dizemos que Sk
é a
 soma se 1/x com x variando de 1 até k. E Ak a área de 1/x com x
variando de
 1 até k. Vemos que tomando os retângulos por baixo da curva 1/x entre
os
 inteiros 1,2 ; 2,3 ;..., 1+Ak>Sk, e os retângulos por cima da curva que
 Sk>Ak+1/(k+1), logo 1+Ak>Sk>Ak+1/(k+1), e como Ak=ln(k), temos
 1+ln(k)>1/1+...+1/k>ln(k)+1/(k+1)
 A diferença entre as pontas dessa desigualdade é 1-1/(k+1)=k/(k+1), de
forma
 que conseguimos estimar Sk com um erro inferior a 1. Era de se esperar
(com
 esse resultado) que (1/1+...+1/k)-ln(k) tendesse a algum número entre 1
e
 1/(k+1), a medida que o k se tornasse maior e de fato  tende para
 0.5772156649... que é a chamada constante de Euler, segundo o Maple V,
e é
 representada pela letra gamma minúscula.
 Já é alguma coisa.
 
 Obrigado!
 
 Eduardo Casagrande Stabel.
 


Apenas uns comentarios:
1) Essa idéia de relacionar as somas com as integrais eh poderosa, eh
padrao e eh 
devida a Mac Laurin.
2) A constante de Euler, ate hoje nao se sabe se ela eh racional ou
irracional.
3)A soma 1+1/2 +...+1/k eh, conforme voce mostrou, aproximada por lnk
com erro 
menor que 1. Tal soma esta tambem relacionada a funçao digama (que
alguns chamam 
de funçao psi), que eh o quociente entre a derivada da funçao gama e a
funçao gama
4) Na realidade, para concluir a existencia da constante de Euler, nao
basta a 
desigualdade que voce mostrou; ha que usar o fato de 1+1/2 +...+1/k -
lnk ser 
monotono, o que eh facil de mostrar.
5) Obrigado a todos pelas dicas sobre o Maple.
Morgado



Re: Pergunta solta

2000-07-30 Por tôpico Augusto Morgado



Marcelo Souza wrote:
> 
> desculpa a ignorância, mas o que é um "maple"
> obrigado
> abraços
> marcelo
> 
> >From: Augusto Morgado <[EMAIL PROTECTED]>
> >Reply-To: [EMAIL PROTECTED]
> >To: [EMAIL PROTECTED]
> >Subject: Re: Pergunta solta
> >Date: Sat, 29 Jul 2000 18:07:57 -0300
> >
> >
> >
> >Ecass Dodebel wrote:
> > >
> > > >From: "Edmilson" <[EMAIL PROTECTED]>
> > > >
> > > >Olá pessoal tudo bem ?
> > > >
> > > >Caro Ecass,
> > > >
> > > >Eu também dei uma olhada no Maple nesta série e fiz assim,
> > > >
> > > >Seja  s(n) = 1/1^2 + ... + 1/n^2 , temos que lim(n->+inf) s(n) = Pi^2/6
> >,
> > > >como s(n) é monótona crescente, temos
> > > >s(n) < Pi^2/6 , para todo n  natural.
> > > >
> > > >Devemos mostrar que  Pi - (6*s(n))^(1/2) < n^(-1) , para todo n
> >natural.
> > > >
> > > >Temos que 2.Pi > n^(-1) , para todo n natural, assim, -2.Pi.n +1 < 0 ,
> > > >completando o quadrado temos :
> > > >
> > > >(n^2)*(Pi^2) - 2.Pi.n +1 < (n^2)*(Pi^2) , ou seja , (n.Pi -1)^2 <
> > > >(n^2)*(Pi^2), assim, como
> > > >
> > > >(n.Pi -1)^2 < (n^2)*(Pi^2) = n².6.Pi²/6 < (n^2)*6*s(n)=6*(n^2)*s(n),
> >temos
> > > >:
> > >
> > > Caro Edmilson,
> > > nesse ponto eu acho que há um pequeno erro, você está fazendo a passagem
> > > n².6.Pi²/6 < (n^2)*6*s(n), ou seja
> > > Pi^2/6 < s(n)
> > > mas lá em cima, você havia dito justamente o contrário, que a s(n) era
> > > inferior a Pi^2/6. Isso invalida a prova, nao?
> > >
> > > Eduardo Casagrande Stabel.
> > >
> > > >
> > > >(n.Pi -1)^2 < 6*(n^2)*s(n), ou seja,
> > > >
> > > >n.Pi - (6*(n^2)*s(n))^(1/2)) < 1 , dividindo ambos os lados por n,
> > > >finalmente :
> > > >
> > > >Pi - (6*s(n))^(1/2)) < n^(-1) , para todo n natural.
> > > >
> > > >Agora sobre o limite do quociente [Pi - (6*s(n))^(1/2))] / [n^(-1)]
> >quando
> > > >n
> > > >tende para o infinito, eu fiz no Maple e este me deu a resposta 3 / Pi
> >que
> > > >está bem próximo de 1.
> > > >
> > > >Esta parte eu deixo para os nossos colegas mostrarem (porque eu ainda
> >não
> > > >consegui).
> > > >
> > > >Atenciosamente,
> > > >Edmilson Aleixo.
> > > >
> > > >- Original Message -
> > > >From: Ecass Dodebel <[EMAIL PROTECTED]>
> > > >To: <[EMAIL PROTECTED]>
> > > >Sent: Saturday, July 29, 2000 3:23 AM
> > > >Subject: Pergunta solta
> > > >
> > > >
> > > > > Olá,
> > > > >
> > > > > Eu tenho uma pergunta meio solta, estava vendo no Maple V a funcao
> > > > >
> > > > > s(n) = 1/1^2 + ... + 1/n^2
> > > > >
> > > > > Sabe-se que lim(n->+inf) s(n) = Pi^2/6, eu estava tentando calcular
> >pi
> > > >por
> > > > > essa funcao, e cheguei a um resultado bem interessante:
> > > > >
> > > > > Pi - (6*s(n))^(1/2) < n^(-1)
> > > > >
> > > > > E também acho que o quociente
> > > > >
> > > > > [Pi - (6*s(n))^(1/2)] / [n^(-1)]
> > > > >
> > > > > tende para 1 quando n tende para o infinito.
> > > > >
> > > > > Não tenho idéia alguma de como provar esses resultados, alguém
> >poderia
> > > >dar
> > > > > uma idéia?
> > > > >
> > > > > Obrigado!
> > > > >
> > > > > Eduardo Casagrande Stabel.
> >
> >Carissimos:
> >A soma dos valores de 1/(n^2) com n variando de k ate infinito sera
> >chamada de Sk.
> >
> >Se voces desenharem a curva y=1/(x^2), pensem no problema de calcular a
> >area Ak entre essa curva e o eixo dos x, a partir (a direita) de x=k.
> >A area esta compreendida entre uma aproximaçao inferior e uma superior
> >obtidas do jeito que vou descrever:
> >Vamos calcular a area de k a infinito como soma das areas de k a k+1,
> >mais a de k+1 a k+2 etc.
> >Na aproximaçao inferior, trocamos a area de k a k+1 pela area de um
> >retangulo obtido trocando nesse intervalo a curva por uma horizontal
> >y=(1/(k+1))

  1   2   >